NCCT Preliminary Exam Flashcards

1
Q

Which of the following actions by the medical assistant is correct when administering medication to a patient sublingually?

A. Placing the medication between the gum and cheek
B. Asking the patient to swallow the medication
C. Applying the medication directly to the skin
D. Placing the medication under the tongue

A

D. Placing the medication under the tongue

Rationale: Sublingual medication is absorbed systemically through intact mucous membranes. Instruct the patient to place the medication under the tongue, sit upright and swallow frequently until the medication dissolves. Ex. Nitroglycerin tablets.

How well did you know this?
1
Not at all
2
3
4
5
Perfectly
2
Q

The medical assistant documents a patient’s temperature as 101.4F. Which of the following types of medication is this patient most likely to receive?

A. Antipyretic
B. Antihistamine
C. Anticoagulant
D. Antiemetic

A

A. Antipyretic

Rationale: Antipyretics are medications used to reduce fevers, and would most likely be used in this scenario. Antihistamines are used to help with allergies, anticoagulants are used to prevent blood clots, and antiemetics are used to prevent nausea and vomiting.

How well did you know this?
1
Not at all
2
3
4
5
Perfectly
3
Q

Which of the following agencies are physicians required to maintain registration with in order to prescribe, dispense, or administer controlled substances?

A. Food and Drug Administration (FDA)
B. Drug Enforcement Administration (DEA)
C. Department of Health and Human Services (DHHS)
D. American Medical Association (AMA)

A

B. Drug Enforcement Administration (DEA)

Rationale: The DEA is tasked with combating drug smuggling and use of illegal drugs within the United States and has sole responsibility for coordination and pursuing US drug investigations abroad. DEA registration certificates must be renewed every 3 years. The FDA is responsible for (in the medical field) protecting and promoting public health through the regulation and supervision of prescription and over-the-counter pharmaceutical drugs (medications), vaccines, biopharmaceuticals, blood transfusions, and medical devices. The DHHS has the goal of protecting the health of all Americans and providing essential human services. The AMA is the largest association of physicians—both MDs and DOs—and medical students in the United States.

How well did you know this?
1
Not at all
2
3
4
5
Perfectly
4
Q

The physician has ordered Benadryl 50 mg p.o. to be given to a patient in the clinic experiencing an allergic reaction. The amount on hand is 25 mg tabs. How many tabs will be administered to the patient?

A. 3 tabs
B. 1.5 tabs
C. 2 tabs
D. 2.5 tabs

A

C. 2 tabs

Rationale: Tabs=25 mg each. Total dose= 50 mg. Total dose= 50/25 per tab= 2 tabs needed. 1 tab is 25 mg, 50=25+25, so need 2 tabs to get a 50 mg dose. 50/X x 25/1, 25X=50, 50/25, X=2 tabs

How well did you know this?
1
Not at all
2
3
4
5
Perfectly
5
Q

The medical assistant notices that the physician prescribes which of the following drugs to patients complaining of frequent insomnia?

A. amLodipine (Norvasc)
B. zolpidem tartrate (Ambien)
C. ondansetron (Zofran)
D. ranitidine (Zantac)

A

B. zolpidem tartrate (Ambien)

Rationale: A common medication to treat insomnia is zolpidem tartrate (Ambien). It is a sedative that works to lessen brain activity and promote sleep. The medication amlodipine (Norvasc) is used to treat hypertension (high blood pressure), as well as angina (chest pain). The drug ondansetron (Zofran) is an antiemetic and is used to treat nausea and vomiting, and ranitidine (Zantac) is used to treat and help prevent heartburn.

How well did you know this?
1
Not at all
2
3
4
5
Perfectly
6
Q

A patient called the office complaining his hydromorphone prescription was not honored at the pharmacy. The patient stated, “The doctor gave me this prescription six weeks ago when I was in the office.” The prescription was denied because it is a schedule

A. I drug.
B. II drug.
C. III drug.
D. IV drug.

A

B. II drug.

Rationale: This is a morphine derivate from an opium extract. Hydromorphone belongs to the group of medicines called narcotic analgesics (pain medicines). It acts on the central nervous system (CNS) to relieve pain. It is classified as a Schedule II drug with a high potential for abuse, therefore, written prescriptions must go to the pharmacy within 72 hours. When a narcotic medicine is used for a long time, it may become habit-forming, causing mental or physical dependence.

How well did you know this?
1
Not at all
2
3
4
5
Perfectly
7
Q

The physician ordered Amoxicillin 500 mg p.o. q.i.d. X 10 days. How many tablets should the pharmacy dispense to fill this prescription?

A. 10
B. 20
C. 30
D. 40

A

D. 40

Rationale: Q.i.d. indicates that the patient will take the medication 4 times per day. Since the patient will take the medication for 10 days, the pharmacy should dispense 40 of the 500 mg tablets. 4x10=40

How well did you know this?
1
Not at all
2
3
4
5
Perfectly
8
Q

The physician orders “Xanax (alprazolam) 0.5mg po tid X 3 days.” Xanax 1 mg scored tablets are available. The medical assistant should write which of the following numbers of tablets on the dispense line for this prescription?

A. 1.5
B. 3
C. 4.5
D. 6

A

C. 4.5

Rationale: To figure out the amount of tablets needed we must use a formula DOSAGE over ON HAND multiply by the vehicle is equal to what to give. So this would be: 0.5mg divided by on hand which in 1mg multiply by the vehicle one tab this would give you a dosage of 0.05mg per dose ( ½ tab) multiply by 3 times a day for 3 days, equal 4.5 tabs to dispense.

How well did you know this?
1
Not at all
2
3
4
5
Perfectly
9
Q

A patient complains of nausea and vomiting, and has been unable to keep fluids down over the course of two days. A suppository has been ordered. Which of the following abbreviations is the correct route of administration?

A. IM
B. PR
C. SC
D. INH

A

B. PR

Rationale: Common pharmaceutical abbreviations are as follows: PR = per rectum, IM = intramuscular, SC = subcutaneous, INH = isoniazid

How well did you know this?
1
Not at all
2
3
4
5
Perfectly
10
Q

Which of the following forms of medication can be administered vaginally?

A. Transdermal and buccal
B. Creams and buccal
C. Creams and suppositories
D. Transdermal and suppositories

A

C. Creams and suppositories

Rationale: Medications are administered vaginally to provide direct vaginal absorption of the medication application and/or insertion. Example: Lotrimin (clotrimazole) is a commonly prescribed antifungal cream or suppository for yeast infection. Transdermal medication is applied directly to the skin to be absorbed systemically (i.e. Fentanyl patch for pain control). Buccal (between the cheek and gums; dental anesthetic) and sublingual (under the tongue; nitroglycerin pill) medications are placed in the mouth for fast absorption.

How well did you know this?
1
Not at all
2
3
4
5
Perfectly
11
Q

A physician wrote an order for the medical assistant to give an insulin injection to an adult patient. Which of the following is the appropriate angle of insertion?

A. 15 degrees
B. 45 degrees
C. 30 degrees
D. 60 degrees

A

B. 45 degrees

Rationale: Insulin is given as a subcutaneous injection. A 45-degree angle is the correct angle of insertion for subcutaneous injections. A 15-degree angle is used for intradermal injections. A 30-degree angle is used for venipuncture and a 60-degree angle is not typically used for injections.

How well did you know this?
1
Not at all
2
3
4
5
Perfectly
12
Q

The patient receiving an albuterol treatment by nebulizer in the clinic complains of tremors, nausea, blurred vision, and headache. Which of the following actions should the medical assistant take?

A. Decrease the oxygen flow meter to three liters per minute.
B. Report these symptoms to the physician.
C. Place the patient on NPO status.
D. Tell the patient these are common side effects.

A

B. Report these symptoms to the physician.

Rationale: Bronchodilators (Ex. Albuterol) relax smooth muscles of the airway. Common side effects are tremors, tachycardia, and hypertension. Nausea, blurred vision, and headache may indicate the patient is having an adverse reaction, or has a toxic level of medication in their system, and should be reported.

How well did you know this?
1
Not at all
2
3
4
5
Perfectly
13
Q

A 6-month-old child is at the clinic for their well-child visit and routine vaccines. Which of the following is the preferred site for an intramuscular injection on this patient?

A. Deltoid
B. Ventrogluteal
C. Dorsogluteal
D. Vastus lateralis

A

D. Vastus lateralis

Rationale: The preferred site for an intramuscular injection on a 6-month old child is the vastus lateralis, or outer thigh, because infants have the most muscle mass in that area. The deltoid muscle isn’t typically used for intramuscular injections until age 3 and older; the ventrogluteal muscle, the hip, isn’t used for infants until 7 months or older. The dorsogluteal muscle, the buttocks, is only used for children 2 and over and adults.

How well did you know this?
1
Not at all
2
3
4
5
Perfectly
14
Q

The physician has ordered acetaminophen (Tylenol) E.S.1000 mg po q day. The amount on hand is 500 mg tabs. What is the correct dose to administer?

A. 2 tabs
B. 1 tab
C. 0.5 tab
D. 2.5 tabs

A

A. 2 tabs

Rationale: 1000/X*500/1, 500X=1000, 1000/500, X=2 tabs

How well did you know this?
1
Not at all
2
3
4
5
Perfectly
15
Q

The medical assistant should stamp checks with the restrictive endorsement “for deposit only” to prevent

A. overdraft fees.
B. nonsufficient funds.
C. stop-payment.
D. loss or theft.

A

D. loss or theft.

Rationale: Endorsement is required to ensure transfer funds from one party to another under the Uniform Negotiable Instrument Act, applicable in all states.

How well did you know this?
1
Not at all
2
3
4
5
Perfectly
16
Q

Upon examination of an adult patient reporting ear pain, the physician discovered a large amount of cerumen in the canal. The order is to irrigate and instill two drops of aminoglycoside (Tobramycin). For the procedure, the medical assistant should pull the

A. earlobe straight up and back.
B. earlobe straight down and forward.
C. pinna up and back.
D. pinna down and back.

A

C. pinna up and back.

Rationale: When doing a procedure on the ear of an adult or child over 3 years of age, the pinna is gently pulled up and back to straighten the ear canal to allow for medication to reach the target area. If the patient is under three years of age, then gently pull back and down.

How well did you know this?
1
Not at all
2
3
4
5
Perfectly
17
Q

The order reads cefaclor (Ceclor) 250 mg p.o. stat, then 150 mg q 2 h. How should the medical assistant interpret this medication order?

A. Give 250 milligrams of cefaclor (Ceclor) orally immediately, and then give 150 milligrams every 2 hours.
B. Give 250 micrograms of cefaclor (Ceclor) orally immediately, and then give 150 micrograms every 2 hours.
C. Give 250 milliliters of cefaclor (Ceclor) post-operatively as needed, and then give 150 milliliters every 2 hours.
D. Give 250 milligrams of cefaclor (Ceclor) post-operatively, and then give 150 milligrams every 2 hours.

A

A. Give 250 milligrams of cefaclor (Ceclor) orally immediately, and then give 150 milligrams every 2 hours.

Rationale: p.o.=oral (by mouth), stat=now (immediately), mg=milligrams, q=every

How well did you know this?
1
Not at all
2
3
4
5
Perfectly
18
Q

Which of the following sites should the medical assistant select to administer an IM injection of antibiotics to an 8-month old infant with bilateral otitis media?

A. deltoid
B. vastus lateralis
C. ventrogluteal
D. dorsogluteal

A

B. vastus lateralis

Rationale: Due to the larger muscle mass, the upper outer thigh is the site of choice for an IM injection until a child has been walking for at least one year. A topical anesthetic (if ordered) can be applied to the injection site prior to injection administration to minimize pain.

How well did you know this?
1
Not at all
2
3
4
5
Perfectly
19
Q

A pediatric patient has been diagnosed with a mild skin infection and the physician orders Amoxicillin. The ordered dose (based on today’s weight) is 91 mg every 8 hours. The patient is to receive a first dose of medication before leaving the office and the suspension available is 125 mg/5 mL. How much medication should the medical assistant administer?

A. 7.2 mg
B. 7.3 mL
C. 3.6 mL
D. 1.8 mg

A

C. 3.6 mL

Rationale: Use the following formula to calculate the proper medication amount: D/H * V where D= desired dose (91 mg), H= dose on hand (125 mg), and V= volume (5mL in this case) 91/125*5=3.64 or set it up as fractions 125 mg / 5 mL = 91 mg / X mL and solve for X 455=125X, X=3.64 mL Since mL is a Volume measurement, 3.64 mL (round to 3.6) of this suspension would give the desired 91 mg dose.

How well did you know this?
1
Not at all
2
3
4
5
Perfectly
20
Q

Which of the following actions is an example of compliance with Quality Control (QC) guidelines for phlebotomy?

A. Run the daily control checks on urinalysis dipsticks.
B. Ensure supplies in venipuncture trays are no more than one month beyond the expiration date.
C. Ensure reusable supplies in venipuncture trays are recycled.
D. Run glucose meter controls between each patient sample.

A

A. Run the daily control checks on urinalysis dipsticks.

Rationale: Proper Quality Control ensures accuracy and reliability of test results while detecting and eliminating errors. It is an important component of patient care to make sure results are accurate, therefore it is important to check controls on urinalysis dipsticks daily. Supplies (in venipuncture trays or anywhere) should not be used beyond the expiration date. Recycling is not a quality control measure. Glucose meter controls should be checked at least daily (or more frequently). It is important to follow whatever quality control protocols are in place for each piece of equipment or manual test performed. Always check to make sure quality control is up to date and within acceptable parameters before running any patient samples.

How well did you know this?
1
Not at all
2
3
4
5
Perfectly
21
Q

Which pregnancy risk drug category indicates the greatest risk of fetal harm?

A. X
B. D
C. C
D. A

A

A. X

Rationale: Medications are categorized to indicate risk of harm to fetus if taken during pregnancy or to nursing mothers. A system of classifying drugs according to their established risks for use during pregnancy includes the following: Category A: Controlled human studies have demonstrated no fetal risk. Category B: Animal studies indicate no fetal risk, but no human studies; or adverse effects in animals, but not in well—controlled human studies. Category C: No adequate human or animal studies; or adverse fetal effects in animal studies, but no available human data. Category D: Evidence of fetal risk, but benefits out-weigh risks. Category X: Evidence of fetal risk. It is thought that risks may outweigh any benefits.

How well did you know this?
1
Not at all
2
3
4
5
Perfectly
22
Q

The medical assistant documented an IM injection as follows: 10/12/XX @ 1030, Rocephin 1 gram IM patient tolerated procedure well. Amy Ray, medical assistant. Which of the following necessary pieces of documentation is missing?

A. Site of administration
B. Physician signature
C. Use of the medication
D. Frequency of administration

A

A. Site of administration

Rationale: Anatomical site of injection should always be documented for reassessment of local and/or systemic reaction. Every intramuscular injection should be documented with date, time, injection site, medication, dosage, person who gave the injection, and record anything relative to the patient’s tolerance of the procedure and response to the medication. Immediately report to the physician any adverse effects from the injection. Physician signature should be on the order, so it isn’t necessarily needed on the injection documentation. Medication uses and frequency should be noted elsewhere on the medical record as well.

How well did you know this?
1
Not at all
2
3
4
5
Perfectly
23
Q

Which of the following solutions is used to fill the reservoir of an autoclave when sterilizing instruments?

A. distilled water
B. normal saline
C. tap water
D. isopropyl alcohol

A

A. distilled water

Rationale: Distilled water does not contain microscopic contaminants, minerals, or calcium found in natural tap water. Distilled water helps destroy microorganisms during the autoclave process and prevents the spread of pathogens. Saline and alcohol would not be used in an autoclave.

How well did you know this?
1
Not at all
2
3
4
5
Perfectly
24
Q

Which of the following is the most common means of transmission of Methicillin-Resistant Staphylococcus Aureus (MRSA)?

A. Direct contact with the skin of infected persons
B. Direct contact with contaminated blood
C. Inhaling contaminated respiratory droplets of infected persons
D. Direct contact with contaminated feces

A

A. Direct contact with the skin of infected persons

Rationale: Because SA can be on the surface of the skin, the most common way MRSA spreads from person to person is by direct contact. A much less common way it can spread is by touching surfaces such as railings, faucets, or handles that may be contaminated with MRSA. Prevention by keeping wounds covered, washing hands regularly, and not sharing personal items such as towels, washcloths, razors, clothing, or uniforms that may have had contact with a contaminated wound or bandage.

How well did you know this?
1
Not at all
2
3
4
5
Perfectly
25
Q

If a medical assistant has questions about disposing of outdated hazardous chemicals, where should the medical assistant look to find this information?

A. HIPAA
B. MSDS
C. CLIA
D. NIOSH

A

B. MSDS

Rationale: A Material Safety Data Sheet (MSDS) is a hazardous chemical reference. It provides healthcare workers with procedures for working with substances safety to ensure regulatory compliance. Clinical Laboratory Improvement Amendments (CLIA) of 1988 are United States federal regulatory standards that apply to all clinical laboratory testing performed on humans in the United States, except clinical trials and basic research. HIPAA (Health Insurance Portability and Accountability Act) requires medical professionals to protect the confidentiality of patients’ health information. The National Institute for Occupational Safety and Health (NIOSH) makes recommendations for preventing illness and injury acquired on the job.

How well did you know this?
1
Not at all
2
3
4
5
Perfectly
26
Q

A patient presents to the clinic reception area with a fever and a harsh cough. Which of the following actions should the medical assistant take? (Select the two (2) correct answers.)

A. ask the patient to wear a mask
B. assist the patient to an exam room for screening
C. test the patient for rapid flu
D. notify the physician immediately
E. prepare the patient for a chest x-ray

A

A. ask the patient to wear a mask
B. assist the patient to an exam room for screening

Rationale: The medical assistant should have the patient wear a mask and assist them to an exam room. These isolation precautions are done to help contain the germs and prevent the spread of infection to anyone else. The other actions do not need to be done immediately.

How well did you know this?
1
Not at all
2
3
4
5
Perfectly
27
Q

For which of the following incidents is employer provided post-exposure follow-up indicated?

A. An employee fall from a step stool while obtaining medical records.
B. A polycup containing urine splashed into the eye of an employee.
C. Loss of power during an ECG procedure.
D. An employee falls in the parking lot, resulting in a hand laceration.

A

B. A polycup containing urine splashed into the eye of an employee.

Rationale: In compliance with OSHA regulations, a post exposure follow-up plan includes incidents of occupational exposure to direct contact with blood or other potentially infectious body fluids. An injury from a fall, whether inside or outside the facility, is not considered an exposure-related injury. Loss of power during any procedure is not related to an exposure, it is an electrical problem (not an injury); no exposure follow-up is indicated in this instance.

How well did you know this?
1
Not at all
2
3
4
5
Perfectly
28
Q

It is appropriate for the medical assistant to use an alcohol-based hand rub as the only method of hand hygiene after which of the following activities?

A. Applying non-sterile gloves for a dressing change
B. Measuring a patient’s apical pulse
C. Using the restroom
D. Assisting the physician with a laceration repair

A

B. Measuring a patient’s apical pulse

Rationale: In addition to when visibly soiled, hands should always be washed with an antimicrobial soap and water before eating and after using a restroom to reduce the incidence of health care environment related infections. Hands must also be washed after assisting the physician with any medical procedures after removing gloves.

How well did you know this?
1
Not at all
2
3
4
5
Perfectly
29
Q

Which of the following diseases is a healthcare worker at the greatest risk of developing following an exposure incident with blood from a needle stick injury?

A. HSV
B. HAV
C. HPV
D. HBV

A

D. HBV

Rationale: Of the options listed, Hepatitis B is the most transmissible infection via needle stick (it is also the only one preventable by vaccination). HSV is Herpes Simplex Virus and is generally transmitted by contact. HAV (Hepatitis A Virus) is most often transmitted as a result of poor hygiene and is spread via fecal-oral contamination. HPV (Human Papilloma Virus) is transmitted by contact (skin to skin or through sexual contact).

How well did you know this?
1
Not at all
2
3
4
5
Perfectly
30
Q

The medical assistant was stuck by a contaminated needle while changing out an overflowing sharps container. The medical assistant should immediately

A. report the incident to a supervisor.
B. wash the exposed area with soap and water.
C. soak the puncture site in BetadineTM.
D. milk the puncture site and clean with alcohol.

A

B. wash the exposed area with soap and water.

Rationale: If a healthcare worker is accidentally stuck with a needle, there is a specific OSHA guidelines to follow: Immediately flush with water, then immediately thereafter tell a supervisor, seek a physician’s care, documents must be filed with date/time of occurrence, patient if known, and type of stick. The source individual should be tested for the following: HBV, HCV and HIV. OSHA requires employee to be notified of the results. The exposed worker needs to be tested for HBV, HCV and HIV. The exposed employee must have a physicians written treatment options within 15 days. NOTE: If the patient source is unknown, the employee will be tested.

How well did you know this?
1
Not at all
2
3
4
5
Perfectly
31
Q

The medical assistant is assisting a physician with an I & D of a gangrenous abscess. In addition to mask and gloves, the medical assistant must wear which of the following personal protective equipment when performing this procedure?

A. Protective eyewear
B. N95 respirator mask
C. Hazmat suit
D. Sterile gown

A

A. Protective eyewear

Rationale: I & D refers to incision and drainage. The staff should be protected from splashes and spurts to prevent transmission of bloodborne pathogens. Proper infection control with personal protective equipment would include masks, gloves, and protective eyewear. This simple medical office procedure would not require the use of a N95 respirator (the infection risk is not airborne), a hazmat suit (there is no hazardous material risk) or sterile gown (this is not considered a sterile surgical procedure).

How well did you know this?
1
Not at all
2
3
4
5
Perfectly
32
Q

Sterilized instruments should be stored

A. for a maximum of 90 days.
B. with the tape side down to avoid susceptibility to contamination.
C. for a maximum of 10 days.
D. in a temperature controlled environment to prevent moisture accumulation.

A

D. in a temperature controlled environment to prevent moisture accumulation.

Rationale: Storage in an environment that controls the temperature maintains the integrity of packaging sterility and prevents the possible infiltration of microorganisms. The date of sterilization and the specific sterilizer used should be clearly indicated on the outside of the packaging material. Storing items within a set time frame used does not insure that the instruments have remained sterile. NOTE: CDC guidelines recommend ensuring consistency of sterilization practices requires a comprehensive program that ensures operator competence and proper methods of cleaning and wrapping instruments, loading the sterilizer, operating the sterilizer, and monitoring of the entire process.

How well did you know this?
1
Not at all
2
3
4
5
Perfectly
33
Q

Which of the following actions should the medical assistant take first after coming in direct contact with a patient’s blood via needlestick injury?

A. Obtain the Hep B vaccine if not currently immunized.
B. Apply pressure at the site to stop bleeding.
C. Flush the area with water and thoroughly wash it.
D. Inform the patient about the incident.

A

C. Flush the area with water and thoroughly wash it.

Rationale: Immediately after a needlestick injury, wash or clean the injury at the needle stick site with soap and water and/or flush mucous membranes. Bleeding could be a good thing, as it serves to help flush pathogens out rather than keep them in the wound. Depending on Hepatitis B vaccination status, it might be necessary to continue the shots (but that would not be the first action after a needlestick). There would be no need to alarm the patient in this case. However, it may be necessary to inform the patient later.

How well did you know this?
1
Not at all
2
3
4
5
Perfectly
34
Q

The medical assistant demonstrates proper handling of sharps containers by doing which of the following?

A. Discard the sharps container in a biohazard waste container when it is two-thirds full.
B. Inspect the sharps container to ensure that only sharps have been deposited.
C. Seal the sharps container and place in the regular waste container.
D. Empty half full sharps containers into a larger container to maintain cost effectiveness.

A

A. Discard the sharps container in a biohazard waste container when it is two-thirds full.

Rationale: Sharps containers hold sharp biohazardous materials and must be sealed and incinerated upon disposal. OSHA (Occupational Safety and Health Administration) mandates that sharps containers be replaced on a routine basis and not be overfilled. Discard sharps containers before they are full to prevent overfilling, which could result in accidental injury or exposure from overflow.

How well did you know this?
1
Not at all
2
3
4
5
Perfectly
35
Q

Which of the following is considered a potentially biohazardous substance?

A. sweat
B. sputum
C. cerumen
D. tears

A

B. sputum

Rationale: A biohazardous substance refers to anything that is contaminated with an agent that could cause an environmental or infectious risk. Sputum is considered a biohazardous substance, because the sputum can be contaminated and harmful to people. Sweat, cerumen, and tears are not generally considered as potentially biohazardous substances because they do not contain contagious pathogen levels (with the exception of severely infectious agents such as Ebola).

How well did you know this?
1
Not at all
2
3
4
5
Perfectly
36
Q

The medical assistant is preparing to remove her gloves after a venipuncture and notices blood on the gloves. Where should the medical assistant dispose of her gloves?

A. Biohazard waste container
B. Red sharps container
C. Regular trash container
D. Specimen transport bag

A

A. Biohazard waste container

Rationale: Biohazard waste includes disposable items with visibly contaminated blood and/or body fluids, therefore the medical assistant should dispose of her gloves in a biohazard container instead of the regular trash (could be an infection risk). A sharps container is for sharp objects (not soft gloves). A specimen transport bag would not be acceptable, because noticeably soiled gloves need to be properly disposed of in a biohazard waste container.

How well did you know this?
1
Not at all
2
3
4
5
Perfectly
37
Q

While emptying the autoclave, the medical assistant notices that the wrapped instruments are damp. The medical assistant should

A. remove the instruments and set them on the counter to dry.
B. remove the instruments and place them on the Mayo stand for surgery.
C. leave the instruments in the autoclave with the door slightly open to finish drying.
D. leave the instruments in the autoclave and run the cycle again.

A

C. leave the instruments in the autoclave with the door slightly open to finish drying.

Rationale: Microorganisms can enter through a moist wrap, which is why it is not advisable to touch or move the contents of an autoclave before they have had sufficient time to dry. Leave all contents in the autoclave and crack the door to allow them to air dry appropriately.

How well did you know this?
1
Not at all
2
3
4
5
Perfectly
38
Q

The medical assistant is preparing an exam room for removal of a benign nevi. Which of the following instrument trays should be set up?

A. ophthalmic tray
B. cryosurgical tray
C. onychectomy tray
D. I & D tray

A

B. cryosurgical tray

Rationale: The medical assistant would need a cryosurgical tray for the removal of the nevi, commonly referred to as moles. Cryosurgery is performed by using extreme cold to destroy the nevi (skin lesions). An ophthalmic tray would be needed if the patient was having a procedure on his/her eyes, and an onychectomy tray would be used for the removal of a fingernail or toenail. An incision and drainage (I&D) would be necessary if the lesion being removed contained drainage or fluid. A nevi would not typically contain drainage or fluid.

How well did you know this?
1
Not at all
2
3
4
5
Perfectly
39
Q

Which of the following actions by the medical assistant is appropriate when interacting with a patient who is visually impaired?

A. Take special care to enunciate each word.
B. Stand behind the patient and gently guide them into the exam room.
C. Explain where things are positioned by referencing a clock image.
D. Limit communication with the patient.

A

C. Explain where things are positioned by referencing a clock image.

Rationale: By providing general orientation to the environment, the visually impaired patient feels more comfortable. It is of utmost importance to maintain a visually impaired patient’s dignity and independence in this type of setting. Guiding them physically takes away independence and could decrease their comfort level. Limiting communication or talking loudly is inappropriate.

How well did you know this?
1
Not at all
2
3
4
5
Perfectly
40
Q

What is the importance of encouraging a patient diagnosed with bronchitis to maintain adequate fluid intake?

A. Open airways
B. Thin secretions
C. Prevents fatigue
D. Decreases inflammation

A

B. Thin secretions

Rationale: Bronchitis is an inflammation of the bronchial tubes, which are in charge of taking air to the lungs. When they get inflamed, thick secretions build up in the bronchial tubes. Patients with bronchitis need to maintain adequate fluid intake in order to thin the secretions they have. Adequate fluid intake will also replace insensible losses and will help to prevent dehydration. By increasing fluid intake and thinning the secretions, the airways will be clearer and there will be less fatigue. Antibiotics are typically prescribed to decrease the inflammation.

How well did you know this?
1
Not at all
2
3
4
5
Perfectly
41
Q

The medical assistant anticipates teaching the patient recovering from a tib-fib fracture about which of the following orthopedic devices?

A. thoraco-lumbo-sacral-orthosis (Boston brace)
B. stabilizing wrist splint
C. cervico-thoraco-lumbo-sacral orthosis (Milwaukee brace)
D. crutches

A

D. crutches

Rationale: The tibia and fibula bones are in the lower leg, therefore the patient would need to be taught about how to properly use crutches to help assist with ambulation. The Boston brace and Milwaukee brace are both used for spine problems, such as scoliosis. The difference between the two is that the Milwaukee brace has a neck ring added to it, and the Boston brace does not. The stabilizing wrist splint is used to hold the wrist in place to prevent further injury.

How well did you know this?
1
Not at all
2
3
4
5
Perfectly
42
Q

During a routine visit, a petite female patient becomes very distraught while discussing the recent death of her husband. She suddenly faints. Which of the following actions should the medical assistant take next?

A. Protect the patient from injury.
B. Activate EMS.
C. Notify the physician.
D. Draw blood for laboratory tests.

A

A. Protect the patient from injury.

Rationale: Protecting the patient is the ultimate priority here. Quickly and carefully lay the patient flat (in the supine position) with her head lower than her feet. Once you have tended to the patient, stay with her to eliminate the potential for any injuries. Then call the physician or other medical personnel to alert them to your situation. Fainting itself does not constitute a reason for activating EMS (that would be necessary if the patient’s heart and/or breathing stopped). Phlebotomy is the last consideration; blood work is not the main concern at this time.

How well did you know this?
1
Not at all
2
3
4
5
Perfectly
43
Q

The patient was seen in the office for follow up evaluation post CVA. The doctor noticed significant weakness on the patient’s left side. Which of the following should the medical assistant expect to perform?

A. Full weight bearing exercise assistance
B. Active assist range of motion
C. Crutch training
D. Pulmonary function test

A

B. Active assist range of motion

Rationale: The medical assistant should expect to perform active assistive range of motion with the patient since the patient has major weakness on one side. This type of exercise is done when a patient is able to perform some movements and exercises with the affected side, but still needs some help to get that side even stronger. Full weight bearing exercises are done to improve bone strength, and are done while the patient is on both feet and putting all of their weight against gravity. This would not be the exercise that this particular patient needs because the patient has weakness on one side. He needs to strengthen that side before he can perform full weight bearing exercises. Crutch training is typically done when a patient is suffering from a leg problem, but since this patient has weakness on his whole left side this would not be appropriate. A pulmonary function test is conducted to help determine risk for stroke, not after a CVA has occurred.

How well did you know this?
1
Not at all
2
3
4
5
Perfectly
44
Q

The medical assistant is obtaining an infant’s respiratory rate. The correct procedure for performing this measurement is to count the respiratory rate for

A. one full minute while observing the rise and fall of the abdomen.
B. one full minute while observing the rise and fall of the chest.
C. 15 seconds and multiply by four while observing the rise and fall of the abdomen.
D. two full minutes while observing the rise and fall of the chest.

A

A. one full minute while observing the rise and fall of the abdomen.

Rationale: Infants and children up to age six are abdominal breathers (not chest breathers like older children and adults). This means that they are still dependent upon diaphragm muscle to breathe. Count an infant’s respiratory rate for one full minute while observing the rise and fall of the abdomen. This is a more accurate measure than counting for less than one minute.

How well did you know this?
1
Not at all
2
3
4
5
Perfectly
45
Q

An adult patient sitting in the waiting area, with a known history of epilepsy, begins having apparent tonic-clonic movement of the arms and legs. How should the medical assistant respond?

A. Provide glucose immediately.
B. Place a tongue depressor in the patient’s mouth to maintain an open airway.
C. Move the patient to the floor and clear surrounding objects.
D. Restrain the patient and hold extremities down.

A

C. Move the patient to the floor and clear surrounding objects.

Rationale: The medical assistant should move the patient to the floor and clear surrounding objects in order to protect the patient from injury. Nothing should be placed in the patient’s mouth, because they could choke on it, and the patient shouldn’t be restrained because this could cause further injury.

How well did you know this?
1
Not at all
2
3
4
5
Perfectly
46
Q

When giving a nebulizer treatment with bronchodilator, the medical assistant should most closely monitor for which of the following complications?

A. Persistent tachycardia
B. Persistent bradycardia
C. Severe nausea and vomiting
D. Severe epistaxis

A

A. Persistent tachycardia

Rationale: The medical assistant should monitor for persistent tachycardia when giving a nebulizer treatment with a bronchodilator. Bronchodilators are medications that open up the bronchioles, and by doing this, the heart rate can increase. Bradycardia (reduced heart rate), nausea and vomiting, and epistaxis (nosebleed) are not typical side effects of pairing a nebulizer treatment with a bronchodilator.

How well did you know this?
1
Not at all
2
3
4
5
Perfectly
47
Q

When collecting a 24-hour urine, a total output of 100 ml for an adult patient is considered

A. oliguria.
B. polyuria.
C. dysuria.
D. enuresis.

A

A. oliguria.

Rationale: A total output of 100 ml of urine for an adult patient would be considered low, and this is referred to as oliguria. Decreased urine output is considered less than 500mL in 24 hours. Polyuria is a large amount of urine output, dysuria is difficult or painful urination, and enuresis is when a person repeatedly cannot control their urination, and can occur at any part of the day.

How well did you know this?
1
Not at all
2
3
4
5
Perfectly
48
Q

The patient is not able to distinguish certain numbers on an Ishihara test. This indicates a problem with which of the following?

A. Rods
B. Cones
C. Pupils
D. Sclera

A

B. Cones

Rationale: The Ishihara test is used to determine color blindness by means of a series of cards each having colored dots that form one pattern to the normal eye and a different pattern to the eye that is color-blind. In the anatomy of the Eye; the cones are responsible for color vision, the rods are responsible for the black and white and night vision. Both the rods and cones are part of the inner layer of the eye. The pupil is the opening in the center of the iris through which light enters the eye. The sclera (from the Greek skleros, meaning hard), also known as the white of the eye, is the opaque, fibrous, protective, outer layer of the eye containing collagen and elastic fiber.

How well did you know this?
1
Not at all
2
3
4
5
Perfectly
49
Q

When performing an Ishihara test, which of the following items is most appropriate for use when tracing the colored dots?

A. pencil
B. dry erase marker
C. cotton swab
D. fingernail

A

C. cotton swab

Rationale: A cotton swab could be used to trace colored dots during an Ishihara vision test. This will help protect the integrity of whatever color vision screening tool is used (plates, a booklet, or on a computer). Fingernails, pencils and dry erase markers could leave marks and skew subsequent results and age the tool. Older Ishihara testing tools contain lead, which is another deterrent for use of the finger to trace the dots. Exposure to lead can cause serious health issues, thus should be avoided.

How well did you know this?
1
Not at all
2
3
4
5
Perfectly
50
Q

After receiving the MMR vaccine, a 12 month old begins to experience coughing, wheezing, and labored breathing. Which of the following actions should the medical assistant take?

A. Initiate chest compressions, followed by rescue breaths.
B. Perform the Heimlich maneuver.
C. Obtain epinephrine and oxygen and await physician’s orders.
D. Call the Poison Control Center immediately.

A

C. Obtain epinephrine and oxygen and await physician’s orders.

Rationale: In the event of an allergic reaction, the medical assistant should obtain epinephrine and oxygen and wait for the physician’s orders. Epinephrine is a medication given during allergic reactions, and oxygen is needed since the child is wheezing and has labored breathing. A shot check should always be performed and documented 20 minutes following an immunization injection to assess for signs of allergic reaction. Chest compressions aren’t necessary since the child can breathe on their own, and the Heimlich is also unnecessary since the child isn’t choking on anything. The Poison Control Center doesn’t need to be called immediately, because treating the allergic reaction and helping the child breathe is the first priority.

How well did you know this?
1
Not at all
2
3
4
5
Perfectly
51
Q

Which of the following complications occurs from prolonged lack of muscle use?

A. Hypertrophy
B. Atrophy
C. Tendonitis
D. Cellulitis

A

B. Atrophy

Rationale: Atrophy, which is the wasting away of a muscle, occurs with prolonged lack of muscle use. This decrease in muscle mass (progressive wasting appearance) is often seen in patients with severe illness, the elderly, or with prolonged immobilization. Hypertrophy is the opposite of atrophy and is the increase in muscle size, tendonitis is the inflammation of the tendons, and cellulitis is a skin infection.

How well did you know this?
1
Not at all
2
3
4
5
Perfectly
52
Q

The medical assistant preps the median cubital vein for a venipuncture procedure. How should the medical assistant prep the patient’s skin?

A. Use back-and-forth friction, applied horizontally or vertically.
B. Apply concentric circular friction, working from the outside inward.
C. Wipe the area lightly from top to bottom in a spiral motion.
D. Wipe the area lightly from bottom to top.

A

A. Use back-and-forth friction, applied horizontally or vertically.

Rationale: A venipuncture is considered a minimally invasive procedure. The fact that the skin will be punctured leaves open the potential to introduce an infection. Therefore, proper care should be taken to minimize the risk of contamination (of the sample) and infection (to the patient). Prep the puncture site with an alcohol pad or equivalent (use a nonalcohol-based cleanser if collecting a blood alcohol level). Use your facility’s prescribed disinfectant for blood culture collections. Proper cleansing is necessary to prevent contamination. Back-and-forth friction has been shown to be more effective than concentric circles. Cleanser should be allowed to air dry before sticking the patient.

How well did you know this?
1
Not at all
2
3
4
5
Perfectly
53
Q

When performing a capillary puncture on an infant, the medical assistant understands the lateral heel is the proper site for collection due to which of the following?

A. The tissue layers on the finger are more susceptible to permanent nerve damage.
B. The finger produces a slower flow of blood, making it more difficult to obtain a specimen.
C. The small amount of tissue between the skin and bone in the finger makes an injury to the bone likely.
D. The blood in the fingertip has been found to contain a higher concentration of hemoglobin than the heel.

A

C. The small amount of tissue between the skin and bone in the finger makes an injury to the bone likely.

Rationale: Heel sticks are easily accessible and minimally invasive for capillary puncture. Fingers of infants are too small to tolerate the trauma of skin puncture.

How well did you know this?
1
Not at all
2
3
4
5
Perfectly
54
Q

The medical assistant applies pressure to the venipuncture site using gauze. After two minutes the medical assistant observes that bleeding has not stopped. The next appropriate action would be to

A. elevate the arm while applying pressure.
B. wrap the arm with a pressure dressing.
C. continue to apply pressure while bending the arm at the elbow.
D. notify the physician immediately.

A

A. elevate the arm while applying pressure.

Rationale: Elevating the arm reduces the pressure from the flow of blood, i.e., the blood is flowing down the arm. The venipuncture wound will heal more easily without the pressure from the blood flow. The arm should not be in a bent position after venipuncture, bending the arm may allow the formation of a hematoma.

How well did you know this?
1
Not at all
2
3
4
5
Perfectly
55
Q

Which of the following blood components are visible in a centrifuged, evacuated red top tube? (Select the two (2) correct answers.)

A. Plasma
B. Serum
C. Individual erythrocytes
D. Platelets
E. Coagulated blood (blood clot)

A

B. Serum
E. Coagulated blood (blood clot)

Rationale: A centrifuged sample in a red top tube will have a liquid portion at the top and a solid portion on the bottom. Since the red top contains no anticoagulant, the blood will clot. This means the top liquid portion is serum and the bottom portion is the coagulated blood. Conversely, centrifugation of an anticoagulated specimen will have plasma as the top liquid part of whole blood, and red cells, white cells and platelets on bottom (no blood clot forms because the specimen has anticoagulant).

How well did you know this?
1
Not at all
2
3
4
5
Perfectly
56
Q

The physician orders the following blood work to evaluate a patient for sepsis: CBC, Blood Culture, and BMP. When using the evacuated tube method for venipuncture, in which of the following orders should the tubes be drawn?

A. Red top, blood culture, lavender top
B. Lavender top, red top, blood culture
C. Blood culture, lavender top, gray top
D. Blood culture, red top, lavender top

A

D. Blood culture, red top, lavender top

Rationale: To assure the accuracy of patient test results, evacuated tubes must be filled in a particular order. Contamination of the blood by the needle entering a stopper or by contact with additives in the tubes can cause erroneous patient test results, leading to improper diagnosis and treatment of the patient. In this case, the Blood Culture should be collected first, followed by the BMP (red top), then the CBC (lavender top).

How well did you know this?
1
Not at all
2
3
4
5
Perfectly
57
Q

Which of the following is the reason for wiping off the first drop of blood from the patient’s finger with gauze when performing a capillary puncture?

A. The first drop may be contaminated by the lancet.
B. The first drop of blood may contain traces of alcohol.
C. The first drop of blood contains more platelets and clots faster.
D. The first drop of blood contains too much oxygen.

A

B. The first drop of blood may contain traces of alcohol.

Rationale: The first drop of blood should be wiped off the finger after the lancet puncture to remove any excess alcohol. The excess alcohol could compromise laboratory results. The first drop contains the same amount of hemoglobin (which carries oxygen in the red blood cells) as the rest of the patient’s blood. Platelet concentration and lancet contamination are not issues in this scenario.

How well did you know this?
1
Not at all
2
3
4
5
Perfectly
58
Q

A medical assistant draws a patient that is on anticoagulant therapy. Which of the following actions should the medical assistant take immediately post venipuncture?

A. Hold pressure over the site and call the nurse
B. Hold pressure and elevate the patients arm
C. Wait for bleeding to stop and apply gauze and tape
D. Apply a pressure dressing over the site

A

A. Hold pressure over the site and call the nurse

Rationale: Complications can arise with any medical procedure. A phlebotomist should confirm hemostasis before leaving a venipuncture patient or allowing the patient to leave. If a patient continues to bleed after a reasonable amount of time, further intervention is required. Another medical professional (like a nurse) may be needed to help stop the bleeding. This is common in patients on anticoagulants.

How well did you know this?
1
Not at all
2
3
4
5
Perfectly
59
Q

Which of the following is the maximum time the medical assistant should leave a tourniquet secured in place on a patient’s arm when performing a venipuncture?

A. 30 seconds
B. three minutes
C. two minutes
D. one minute

A

D. one minute

Rationale: The advised time limit for having a tourniquet applied to an arm is one minute. This limit is in place to prevent flow constriction and increased risk of clotting during the venipuncture. Hemoconcentration, which may affect test results, can occur if the tourniquet is left on for more than one minute.

How well did you know this?
1
Not at all
2
3
4
5
Perfectly
60
Q

A patient shares with the medical assistant the last time he was in the clinic to have his blood drawn, he “became very dizzy and felt like the room was spinning.” Which of the following actions should the medical assistant take?

A. Ask the patient to lean slightly forward during the venipuncture.
B. Reassure the patient the procedure will be over before dizziness occurs.
C. Encourage the patient to lie down during the venipuncture.
D. Encourage the patient to sit upright on the exam table during the venipuncture.

A

C. Encourage the patient to lie down during the venipuncture.

Rationale: The medical assistant should encourage the patient to lie down during the venipuncture. This will help to prevent syncope and maintain safety for the patient. Having the patient sitting up during the procedure can cause them to become dizzy and lightheaded since the patient is losing blood, but patients who are lying down rarely faint during the venipuncture.

How well did you know this?
1
Not at all
2
3
4
5
Perfectly
61
Q

An older adult patient arrives to have blood drawn. While assessing the arms for venipuncture, the medical assistant notices scar tissue in the anticubital areas and is unable to palpate a vein. Which of the following is an appropriate alternative course of action?

A. Draw blood from the hand using an evacuated tube system.
B. Draw blood from the hand using a winged infusion set.
C. Use a larger gauge needle to penetrate through the scar tissue.
D. Perform a capillary puncture on the lateral thumb.

A

B. Draw blood from the hand using a winged infusion set.

Rationale: Areas with visible scar tissue should be avoided when selecting a site for venipuncture. Scarring can cause the area to be difficult to puncture and may obstruct blood flow. So, the best approach is to draw blood from the hand using a winged infusion set (a.k.a. butterfly). Use of an evacuated tube could collapse the vein due to the vacuum pressure. A capillary puncture would not be the best approach because a venous draw on the hand would give a superior specimen.

How well did you know this?
1
Not at all
2
3
4
5
Perfectly
62
Q

The rubber sleeve at the end of a venipuncture needle allows for

A. single tube collections.
B. slower draw of the specimen.
C. multiple tube collections.
D. faster draw of the specimen.

A

C. multiple tube collections.

Rationale: The rubber “re-seals” the needle and maintains a closed system, thus allowing multiple tubes to be drawn. The rate of flow is independent of that sleeve. Flow rate depends on several factors, including patient blood pressure, size of needle, etc.

How well did you know this?
1
Not at all
2
3
4
5
Perfectly
63
Q

A physician has ordered STAT blood work for a patient. The tests ordered are Hgb/Hct, chemistry panel, and PT test. Which of the following tubes are needed for this procedure? (Select the three (3) tubes that apply.)

A. lavender
B. gray
C. light blue
D. red
E. yellow

A

A. lavender
C. light blue
D. red

Rationale: The medical assistant should draw the following tubes: Lavender for the Hgb, Hct Red for serum chemistry test Light blue for the PT (Yellow is used for blood cultures and gray is generally used for glucose samples.) CLSI lists the order of draw as follows. 1. Blood culture tube 2. Coagulation tube (blue closure) 3. Serum tube with or without clot activator, with or without gel (red closure) 4. Heparin tube with or without gel plasma separator (green closure) 5. EDTA tube with or without gel separator (lavender closure, pearl closure) 6. Glycolytic inhibitor (gray closure)

64
Q

Which of the following conditions may occur if a tourniquet is left in place on a patient’s extremity for longer than 60 seconds?

A. Venous stasis
B. Petechiae
C. Hemolytic anemia
D. Hemangioma

A

A. Venous stasis

Rationale: A tourniquet left in place for longer than 60 seconds predisposes the formation of blood clots due to slow blood flow (a.k.a. venous stasis). Petechiae are pinpoint spots on the skin that appear when the capillaries break and release blood. Hemolytic anemia refers to a condition defined by red blood cell destruction, which leads to an overall reduction of healthy red blood cells left to circulate and deliver oxygen throughout the body. Hemangioma is a term used to describe a condition in which blood vessels abnormally congregate in a place on the body, causing a non-cancerous tumor.

65
Q

When inspecting a patient’s arm before performing a venipuncture, the most desirable site appears to be the back of the hand. Which of the following venipuncture methods is most appropriate in this situation?

A. Evacuated tube
B. Needle and syringe
C. Butterfly needle
D. Capillary puncture

A

C. Butterfly needle

Rationale: The best choice in this situation is to use a butterfly needle, a winged needle with flexible tubing that is short in length. Butterfly needles are easier to insert into tiny, fragile, and/or rolling surface veins close to the skin. If a butterfly is not available, a needle and syringe would be the next choice. The vacuum pressure of evacuated tubes can collapse small or fragile veins such as those on the back of the hand. A capillary puncture would be the last option here, and would be used in the instance of a failed butterfly or syringe attempt (provided there are no other veins available).

66
Q

What does the color of an evacuated collection tube represent?

A. A visual identifier for tests expected to be above or below normal range.
B. The minimum volume of blood to be obtained.
C. The personal protective equipment required for collection.
D. The type or absence of an additive.

A

D. The type or absence of an additive.

Rationale: There are a variety of blood collection tubes. The colors of the caps indicate the presence or absence of additive. Particular laboratory tests may require a specific additive. For instance, if a test requires serum, a red top tube that allows the blood to clot would suffice. A lavender/purple top tube contains K3 EDTA anticoagulant that will keep blood from clotting, allowing for whole blood testing such as a CBC and Differential. Depending upon the type of tube and additive therein, a minimum blood volume may be required.

67
Q

Which of the following actions should the medical assistant take when disposing of a needle following a venipuncture?

A. Recap the needle and place it immediately in a sharps container.
B. Push the needle guard over the needle until it clicks and then place in a biohazard bag.
C. Place the needle with safety device activated immediately in a sharps container.
D. Recap the needle and place it on the blood collection tray.

A

C. Place the needle with safety device activated immediately in a sharps container.

Rationale: To minimize risk of needlestick exposure and/or injury, the medical assistant should place the needle with safety device activated immediately in a sharps container. Recapping the needle increases the risk of an accidental needlestick and is not recommended (if the needle has a needle guard, it should be activated to minimize risk). All needles should be promptly disposed of in a sharps container, not in a biohazard bag or on the blood collection tray.

68
Q

The physician orders a random blood sugar on an adult patient newly diagnosed with diabetes who is right hand dominant. The medical assistant notices inappropriate puncture markings on the patient’s fingers. Which of the following statements should the medical assistant use when re-educating the patient in capillary puncture?

A. “It’s best to use the outer edge of the middle or ring finger on your left hand when doing your finger stick.”
B. “You should consider using the center of the middle finger on either hand for your blood sugar testing.”
C. “Do not use your right hand when checking your blood sugar.”
D. “It’s best to use the outer edge of the under-side of your big toe when performing capillary punctures.”

A

A. “It’s best to use the outer edge of the middle or ring finger on your left hand when doing your finger stick.”

Rationale: Regular blood sugar testing is an important tool for managing diabetes, and newly diagnosed diabetics need guidance on how best to obtain accurate blood sugar readings on their glucose meter. Capillary blood from the fingertips is one of the best sources from which to obtain results that are a true reflection of the blood sugar status. Alternative site testing (forearm, palm, stomach, and thigh) can also be utilized, however the glucose readings may not be a true reflection of current sugar levels. For example, it could take 20 to 30 minutes before the blood glucose reading from the fingertip reaches the forearm. Patients are generally instructed to utilize the middle or ring finger of the non-dominant hand. The sides of the finger have better blood flow and are not as painful as the pad of the finger. Therefore, the medical assistant should instruct the patient in this instance to use the outer edge of the middle or ring finger on the left hand.

69
Q

Which of the following must be written on a patient’s label when handling a specimen tube after it has been obtained?

A. medical assistant’s initials
B. ordered test
C. physician’s name
D. patient’s gender

A

A. medical assistant’s initials

Rationale: It is important to have a time, date, and initials on a tube label to ensure accuracy. The physician’s name, ordered tests, and patient gender should all be on the testing requisition. Depending on the standard collection protocol of the medical practice, it may be required to label the tube with a sticker (that may or may not be pre-printed). It is important that the sample can be traced to the person who collected it (hence the initials), in addition to the time and date of collection.

70
Q

A CMP is ordered for an older adult patient whose veins continue to collapse despite drinking water. Which of the following is the best collection method for this patient?

A. Evacuated tube system
B. Needle and syringe
C. Capillary puncture
D. Butterfly needle

A

D. Butterfly needle

Rationale: A butterfly is a smaller gauge needle device and is used for smaller veins, including fragile veins on all patients, elderly adults and small children. Other phlebotomy systems use larger gauge needles. The vacuum pressure in an evacuated tube system would collapse the vein. A needle and syringe would also have a tendency to collapse the vein (when the plunger of the syringe is pulled back). A capillary collection would be the last resort if all other collection methods failed. The butterfly is the best initial choice in this case.

71
Q

A finger stick capillary puncture should be performed at which of the following sites to prevent injury?

A. At the most distal end of the fingertip
B. Slightly to the side of the center of the fingertip
C. Directly in the center of the fingertip
D. Proximal to the skin crease located at the first joint on the fingertip

A

B. Slightly to the side of the center of the fingertip

Rationale: Capillary blood from the finger is one of the best sources from which to obtain certain samples, such as those needed for blood sugar readings. To ensure adequate blood flow and prevent injury, the proper approach is to utilize the middle or ring finger of the patient’s non-dominant hand; the sides of the finger have better blood flow and are not as painful as the pad (center) or the actual distal end of the finger. The skin crease at the first joint would not provide adequate blood and could cause more pain and/or injury, therefore this area should be avoided during a finger stick.

72
Q

Which of the following techniques is most likely to cause contamination of a urine specimen during collection?

A. retracting uncircumcised foreskin prior to cleansing with sterile wipes
B. cleansing around the foreskin in a circular motion prior to collection
C. cleansing the urethral meatus from back to front with sterile wipes
D. cleansing the urethral meatus from front to back with sterile wipes

A

C. cleansing the urethral meatus from back to front with sterile wipes

Rationale: Cleansing the urethral meatus from the back to front with sterile wipes would cause contamination of the urine specimen, because fecal contaminants from the rectal area could be brought to the urethra and cause an infection. The patient should always be instructed to wipe from front to back, and the other two options are correct ways to prevent contamination.

73
Q

Which of the following laboratory specimens is acceptable for a patient to collect at home with proper education?

A. urine specimen for C&S
B. random sample urine drug screen
C. wound culture specimen
D. fecal specimen

A

D. fecal specimen

Rationale: With proper education, it is acceptable to allow fecal collections to occur at home. For obvious reasons, this helps facilitate collection timing and yields adequate amount of stool specimen. The medical professional should instruct then ask the patient to verbalize understanding of the collection process to ensure quality and accuracy. A drug screen needs to follow a chain of custody, and therefore cannot be collected solely by the patient at home. Any culture, whether it be on the urine or in a wound, should be collected in a sterile container or swab, either by or under the supervision of a medical professional.

74
Q

Which of the following is a CLIA waived test that can be performed by the medical assistant?

A. Differential
B. Chemical urinalysis
C. Gram stain analysis
D. Crossmatch

A

B. Chemical urinalysis

Rationale: A medical assistant can perform a chemical urinalysis test, which shows if the patient has protein or glucose in their urine, but would require intervention by the primary care provider. A chemical urinalysis can also tell what the specific gravity of the urine, meaning how diluted or concentrated it is. The test is done with a small dipstick that changes color when it comes into contact with urine. The medical assistant then compares the stick to a chart to see what is in the urine and if anything is abnormal. A differential is done by a laboratory technician. It is done by drawing blood and then placing the blood on a glass slide to look for any signs of infection or anemia. A Gram stain analysis is also done in the lab under a microscope when a bacterial or viral infection is suspected. Cross-matching is a test administered before a blood transfusion to ensure the donor’s blood is compatible with the recipient. This test is performed by a certified laboratory technologist.

75
Q

When preparing to assist the physician with collection of a deep wound culture, the medical assistant should have which of the following available?

A. Blood culture media
B. Viral transport media
C. Bacterial culture swabs
D. Cotton tipped swabs

A

C. Bacterial culture swabs

Rationale: When helping with a deep wound culture, the medical assistant needs to have bacterial culture swabs available in order to try to identify what bacteria is causing the infection. Bacteria causing infection in a deep wound would likely be anaerobic (because oxygen is not prevalent in a deep wound). For this reason, it is important to make sure the bacterial swab is collected and transported in the appropriate swab system to sustain anaerobic bacteria. Blood culture media is taken by performing a venipuncture to see if an infection is present. Since the physician is looking for a bacterial infection, the viral transport media would not be used, and cotton tipped swabs would not be the appropriate swab to use since the goal is to determine what kind of bacteria is causing the infection.

76
Q

The medical assistant is providing instructions for a patient with pneumonia. The optimal time a sputum specimen should be collected is

A. first thing in the morning.
B. after lunch.
C. at bedtime.
D. after breakfast.

A

A. first thing in the morning.

Rationale: Sputum is the phlegm or mucus from the lungs, and is not to be confused with regular saliva or ‘snot’ that drains from the nose. Sputum needs to be ‘coughed up’ from the lungs. When collecting a specimen for laboratory testing, optimal timing is important to ensure that the best possible specimen is submitted. The production of sputum is the greatest first thing in the morning. The sputum contains expectorated overnight secretions and a first morning sample generally will have the highest concentrated amount of whatever pathogen is causing the pneumonia. The sample should be collected before the patient eats or drinks anything.

77
Q

A patient with the left arm amputated comes in for an ECG. Which of the following is the proper placement for the left arm lead on this patient?

A. the same as it would be for a patient without a left arm amputated
B. moved to the back on the side of the left shoulder moving the right lead to match
C. as close to the site as possible but higher on the limb and in a similar position
D. moved to the back side of the left shoulder and in a similar position

A

C. as close to the site as possible but higher on the limb and in a similar position

Rationale: The person performing the ECG should place the limb leads in close proximity as he/she normally would. In this case of the left arm being amputated, the lead would be placed on the upper portion of the arm or on the front part of the shoulder, depending on where the arm was amputated.

78
Q

A female patient has come in for an ECG following a mastectomy. ECG electrodes should be placed

A. three inches above the surgery site to avoid disruption of the site.
B. as close to the surgery site as possible without touching the site.
C. three inches lower than the surgery site in order to avoid disruption of the site.
D. in the intercostal spaces above the heart to avoid the surgical site.

A

B. as close to the surgery site as possible without touching the site.

Rationale: If a female patient needing an ECG has had a mastectomy, the person performing the ECG should place the electrodes as close to the area they would normally be placed without touching the site of surgery. If that person were to place the electrodes three inches above or below the surgery site, the electrodes would be too far away from the correct placement and therefore could give false recordings.

79
Q

Male patients are properly prepared and positioned for an ECG if they are positioned in

A. Sims with chest and extremities exposed.
B. supine with chest and extremities exposed.
C. supine with chest exposed.
D. Sims with chest exposed.

A

B. supine with chest and extremities exposed.

Rationale: Considering the nature of ECG testing, patients need to be in the supine position with their chest and extremities exposed. The person performing the ECG should exercise discretion for female patients in this position. Supine positioning refers to a position in which the patient is lying face up. A patient lying in the Sims position is generally on their side with the lower leg straight and upper leg bent at the knee.

80
Q

The patient is performing a cardiac stress test. Which of the following is the proper placement for lead V2?

A. fourth intercostal area to the right of the sternum
B. fourth intercostal area to the left of the sternum
C. mid clavicular at the fifth intercostal
D. mid axillary line horizontal to V3

A

B. fourth intercostal area to the left of the sternum

Rationale: In order to capture the appropriate ECG tracings during a cardiac stress test, the V2 lead should be placed at the fourth intercostal area to the left of the sternum.

81
Q

The use of smaller electrodes while running an ECG on a four-year-old pediatric patient will require which of the following modifications?

A. Decrease the paper speed.
B. Increase the paper speed.
C. Place the patient in Sim’s position.
D. Place the patient in a dorsal recumbent position.

A

B. Increase the paper speed.

Rationale: Any adjustment to the ECG instrument should be noted on the recording to avoid erroneous interpretation. The paper speed should be increased on pediatric patients (who have faster heart rates) so the physician can view each deflection on the tracing. An ECG on an infant or child is generally performed when the patient is in the supine position. Occasionally, a pediatric patient will become restless or distressed, in which case the ECG can be performed from a sitting position.

82
Q

Which of the following actions should the medical assistant take to ensure proper monitoring of a patient during a stress test? (Select the three (3) correct answers.)

A. Instruct the patient to start at a rapid pace.
B. Instruct the patient to report any symptoms.
C. Adjust the temperature of the room to the patient’s preference.
D. Measure blood pressure and heart rate frequently.
E. Observe facial expressions, skin color and perspiration.

A

B. Instruct the patient to report any symptoms.
D. Measure blood pressure and heart rate frequently.
E. Observe facial expressions, skin color and perspiration.

Rationale: Instructing the patient to report any symptoms, measuring blood pressure and heart rate, and observing facial expressions, skin color, and perspiration are all appropriate actions for the person performing the ECG to monitor while performing a stress test. Although regulating room temperature would be convenient, the person performing the ECG may not have the ability to do so. The temperature of the room is often automatically controlled. During a stress test, the patient starts out at a walk and then the pace is later increased (the patient does not start at a rapid pace).

83
Q

Which of the following ECG leads records the electrical activity of the inferior surface of the left ventricle and is closest to the heart?

A. Lead I
B. aVR
C. Lead II
D. aVL

A

C. Lead II

Rationale: Lead II records tracings between the right arm and left leg. This is the lead recorded on a rhythm strip at the bottom of a 12 lead ECG.

84
Q

While performing an ECG on a patient, electrodes keep falling off. Which of the following should the medical assistant do? (Select the two (2) correct answers).

A. Place tape over the wires of the electrodes.
B. Clean the skin with soap and water.
C. Clean the skin with 70% isopropyl alcohol.
D. Reposition the electrode.
E. Ask the patient to sit on her hands.

A

A. Place tape over the wires of the electrodes.
C. Clean the skin with 70% isopropyl alcohol.

Rationale: Since the electrodes will not stay on the patient, the person performing the ECG should clean the skin with 70% isopropyl alcohol. Tape should not be placed directly over the electrodes, but on the wires if necessary. Iodine is normally used to disinfect skin before surgical procedures and would not be the best option for getting electrodes to stick to skin. Doubling the electrode patches would also not be the best choice in this scenario because the electrodes are not sticking because the skin is oily or dirty.

85
Q

A patient comes into the office for an ECG. The medical assistant notices the patient has a burn that covers the patients’ entire right arm. Which of the following is the proper lead placement?

A. Place the V1 - V6 leads on the chest, RL and LL leads on the legs, RA and LA leads on the arms.
B. Place the V1-V6 leads on the chest, RL and LA leads on the lower abdomen, RA and LA leads on the chest
C. Place the V1-V6 leads on the chest, RL and LL leads on the legs, RA and LA leads just below the clavicle.
D. Place the V1-V6 leads on the chest, RL and LL leads on the legs, RA and LA leads on the abdomen

A

C. Place the V1-V6 leads on the chest, RL and LL leads on the legs, RA and LA leads just below the clavicle.

Rationale: The correct placement for the arms leads is usually the upper, outer, fleshy part of the arms. However, alternate placement such as the wrist, shoulder and upper chest may be used in certain situations. If the patient has excessive scar tissue, an open wound, excessive hair, a cast or an amputated limb, the alternate placement is acceptable.

86
Q

Which of the following should be done to correct a wandering baseline artifact?

A. Remind the patient to remain still.
B. Keep lead wires uncrossed.
C. Ask the patient to turn off his cell phone.
D. Unplug other electrical appliances in room.

A

A. Remind the patient to remain still.

Rationale: Causes of a wandering baseline may include poor skin preparation, loose electrodes, old electrodes, incorrect placement of the electrodes, and movement.

87
Q

To correct a wandering baseline, the medical assistant should

A. unplug the electrical equipment around the ECG.
B. ask the patient to remove jewelry.
C. replace loose electrodes.
D. replace broken cables.

A

C. replace loose electrodes.

Rationale: While all these can cause artifact, a wondering baseline is due to a loose electrode (the electrode is partially touching the skin and causing the baseline to wander back and forth). Electrical equipment around the ECG device can cause 60 cycle interference. A broken cable will cause a drop out in the waveforms depending on which cable. Patient movement can cause static on the tracing.

88
Q

A parent calls the pediatrician’s office very upset because her child is vomiting and has a temperature of 103.3 F. The medical assistant should

A. schedule a same day work-in appointment.
B. place the parent on hold and transfer the call to the physician.
C. instruct the parent to take the child to the emergency room immediately.
D. suggest an over the counter anti-pyretic.

A

A. schedule a same day work-in appointment.

Rationale: A child presenting with a high fever is an acute situation. In this case, it is best to work the child in for an appointment that same day. Treatment should be started as soon as possible to minimize the side effects brought on by a high fever.

89
Q

A patient presents to the medical office of the primary care provider stating the following: “I was previously seen for physical exams, and the EOB I received contained an error regarding my medical coverage. The doctor drew blood screening for hypercholesterolemia, and I’m out of my blood pressure meds.” Which of the following questions should the medical assistant ask the patient first?

A. Were you referred by a physician?
B. Did you receive your EOB for that visit?
C. Will you be paying for the bill?
D. Did the doctor say you needed to be seen today?

A

D. Did the doctor say you needed to be seen today?

Rationale: Front office patient triage can be difficult. It is important to determine how urgently patients may need to be seen by the doctor. If the doctor did not instruct the patient to return the same day, then the patient’s payment error and prescription request could be handled in a non-urgent manner. The patient may need to work directly with the biller if the EOB contained an error, or verification of services can be made by the medial office assistant for that particular episode-of-care. A prescription request can be prepared and presented to the provider at a later time. Providers usually have a designated time of the day which they manually or electronically authorize prescriptions.

90
Q

A patient’s wife calls the office and states that her husband is experiencing left arm pain and chest pressure. Which of the following could the patient be suffering from?

A. IBS
B. CAD
C. CVA
D. MI

A

D. MI

Rationale: Left arm pain and chest pressure can be some of the warning signs of Myocardial Infarction (MI). Irritable Bowel Syndrome (IBS), Coronary Artery Disease (CAD) and a Cerebrovascular Accident (CVA) have symptoms that present differently than MI. Getting the patient to immediate care is essential in a case such as this for best treatment.

91
Q

Which of the following statements support the purpose of taking extra care in using appropriate grammar in written medical office communication? (Select the three (3) correct answers.)

A. It is a reflection on the physician and medical practice.
B. It makes a positive impression on others.
C. It demonstrates professionalism and competence.
D. It initiates a contractual agreement with the patients.
E. It safeguards protected health information.

A

A. It is a reflection on the physician and medical practice.
B. It makes a positive impression on others.
C. It demonstrates professionalism and competence.

Rationale: Use of proper grammar will reflect positively on the entire staff within a medical practice. This leaves a good impression on patients and visitors as it serves to demonstrate overall competence and professionalism. This in turn can help patients feel more at ease believing they are getting competent care. Use of proper grammar does not initiate contractual agreements or safeguard PHI (protected health information).

92
Q

Which of the following USPS mailing services should the medical assistant select when mailing routine patient records to an insurance carrier?

A. standard mail
B. priority mail
C. media mail
D. registered mail

A

A. standard mail

Rationale: Standard mail is generally adequate for sending patient records to an insurance carrier. Mail in this class must weigh less than 15 lbs. www.usps.com explains business mail options http://pe.usps.com/businessmail101/classes/packageServices.htm

93
Q

A patient with diabetes has missed all scheduled appointments in the past 18 months. The physician requests that written notification be sent to the patient to make an appointment immediately. This letter should be sent

A. priority.
B. certified.
C. express.
D. standard.

A

B. certified.

Rationale: The letter to the patient should be sent Certified so that the physician can see when the patient receives the letter. With Certified mail, the patient must sign for the letter when he/she receives it. This ensures that it was delivered to the correct person. Since the patient has diabetes, it is important for him/her to see a physician regularly to prevent the disease from getting worse. If a letter is sent Priority it will ship faster than general or first-class mail but would not be the best choice because the medical assistant needs to make sure that the patient received the letter about his/her upcoming appointment. Express mail is an international service that allows people to send mail from country to country and would not be needed in this case because the patient and physician are in the same country. Standard mail is not sent first-class and items typically sent by standard mail are newspapers, flyers, and magazines. This also would not be the best choice because the medical assistant would not have confirmation that the patient received the letter.

94
Q

While scheduling an appointment, the medical assistant’s computer freezes. When attempting to restart the program, which of the following should the medical office assistant do first?

A. Unplug computer from the outlet source.
B. Press the Ctrl, Alt and Delete keys.
C. Shut down the computer.
D. Restart the computer.

A

B. Press the Ctrl, Alt and Delete keys.

Rationale: By pressing the CTRL, ALT and Delete keys, at one time, the computer can reboot by shutting down and then restarting. By pulling the power cord, you can run the risk of damaging the files and hardware.

95
Q

Flow charts, progress notes, and narrative notation are all examples of

A. medical record charting styles.
B. patient chart filing systems.
C. recording patient information in a medical record.
D. patient documentation in a SOAP format.

A

C. recording patient information in a medical record.

Rationale: Patient information can be recorded in the medical record in a variety of ways. Progress notes, flow charts and narrative notation are all effective ways to document a patient’s medical information. The SOAP format is the documentation system chosen by many physicians. S=Subjective patient interview information, O=Objective information such as vitals, lab values, etc., A=Assessment of the “S” and “O” data, P=Plan for treatment or follow-up. Medical records of various styles are generally kept in charts, which are filed according to a filing system.

96
Q

Which of the following forms should always be included when sending a fax originating from the medical office?

A. release of information
B. HIPAA waiver
C. cover sheet
D. informed consent

A

C. cover sheet

Rationale: A cover sheet should be included with all outgoing faxes from a medical office. It provides the information necessary for indicating direction and purpose of the attached documents. The cover sheet also protects office confidentiality. Releases, waivers and consent forms are not always part of a communication via fax.

97
Q

The transfer of information such as patient eligibility, payment remittances, claims, claim status, authorizations and referrals are addressed under the HIPAA Administrative Simplification and Compliance Act as electronic

A. report management.
B. remittance advice.
C. data interchange.
D. clearinghouse.

A

C. data interchange.

Rationale: Data Interchange is defined as the automated transfer of data between a care provider and payer. The benefits to care providers include quicker turnaround of information, reducing administrative expenses and avoiding claim processing delays. HIPAA included Administrative Simplification provisions that required HHS to adopt national standards for electronic health care transactions and code sets, unique health identifiers, and security. At the same time, Congress recognized that advances in electronic technology and data interchange could erode the privacy of health information.

98
Q

Which of the following is the most appropriate action when there are delays in the physician’s schedule?

A. Notify patients when there will be more than a 45-minute wait.
B. Notify patients when there is an emergency only, because physician will catch up.
C. Offer to cancel and let them call for another appointment.
D. Give patients an estimate of delay and options to deal with delay.

A

D. Give patients an estimate of delay and options to deal with delay.

Rationale: It is highly common for delays to occur in any medical practice, and it is best practice to keep patients informed of any delays they may experience at the office (regardless of the reason). Patients should be given an estimated delay time and options to deal with the delay. They may choose to wait or reschedule, depending on the expected wait time.

99
Q

Which of the following types of appointment scheduling would be the best method for an office with standard hours, multiple practitioners, and accommodating work-in appointments?

A. Clustering
B. Open office hours
C. Double booking
D. Wave

A

D. Wave

Rationale: Since the office has standard hours and multiple practitioners, the wave scheduling would provide the best outcome. This type of scheduling works best when there is sufficient staff and multiple rooms for patients to be seen in. Wave scheduling is done by scheduling a few patients at the top and bottom of each hour. The first patient to arrive is typically seen first, unless there is a patient with a greater health concern. This allows all of the practitioners to see patients, and provides adequate time for walk-in patients to be seen. The clustering form of scheduling does not allow much time for walk-in appointments as multiple patients with similar problems are scheduled at the same time of day. Stream scheduling is typically used when only one practitioner is available as one patient is assigned to a certain time. Double booking is also typically used when only one practitioner is available; multiple patients are scheduled for the same time. Open office hours lump all patients as walk-ins and they are seen in the order in which they check in. This can lead to extensive wait times if several patients come at the same time.

100
Q

The medical office assistant is scheduling new patients for an office with three physicians and two nurse practitioners. The office policy is for patients to arrive 30 to 45 minutes prior to their appointment to complete patient history forms. Which of the following is the most efficient scheduling technique for this office?

A. time-specified scheduling
B. wave scheduling
C. modified wave scheduling
D. open booking

A

A. time-specified scheduling

Rationale: Time-specified scheduling is the most efficient method of scheduling in this situation. Wave, modified wave and open booking would not be the best choice to optimize the scheduling of new patients.

101
Q

Which type of scheduling works well for specialty and consulting practices because it allows the physician time to prepare for each office visit with the knowledge that the day will begin and end on schedule if patients adhere to their assigned times?

A. stream scheduling
B. double booking
C. triage scheduling
D. open hours scheduling

A

A. stream scheduling

Rationale: The best type of scheduling for this scenario would be stream scheduling since each patient is assigned an individual time to be seen. Double booking involves booking more than one patient to the same appointment time. Scheduling patients due to the seriousness of their situation is an example of triage scheduling. Open hours scheduling allows patients to be seen in the order in which they arrive at the clinic.

102
Q

When managing multiple incoming calls, the medical assistant should handle which of the following calls first?

A. A patient calling to schedule a follow-up visit for uncontrolled pain after hip surgery.
B. A patient calling to cancel today’s appointment for a needle biopsy of a breast mass.
C. A lawyer with a subpoena asking for patient information.
D. A neurologist seeing a patient on consult.

A

D. A neurologist seeing a patient on consult.

Rationale: The medical assistant’s main priority is to handle the most urgent call promptly. A neurologist calling to report on a referred patient would take priority over anyone scheduling or cancelling an appointment. The other calls can be put on hold or they can opt to have their names and phone numbers taken for calls to be returned and handled when time permits. A lawyer with a subpoena can also wait for a return call until all other priorities have been handled.

103
Q

The medical assistant is scheduling a patient for a gallbladder study. Which of the following terms should the medical assistant use on the patient test order?

A. Angiography
B. Cystography
C. Cholecystography
D. Cholangiography

A

C. Cholecystography

Rationale: The suffix “graphy” refers to writing, recording or otherwise representing something (data, image, etc.). Angio= blood or vessel Cysto= cyst, urinary bladder or sack of fluid Chole= gall or bile Therefore, a gallbladder study would include a cholecystography.

104
Q

Fines ranging from $100 to $50,000 may be imposed to an individual for a violation of which of the following?

A. OSHA standards
B. CLIA
C. HIPAA
D. CDC guidelines

A

C. HIPAA

Rationale: The HIPAA Privacy Act- penalties emphasize the importance of compliance. Information in medical records is considered highly private and sensitive. Medical ethics rules, state laws, and the federal law, generally require doctors and their staff to keep patients’ medical records confidential, unless the patient allows the doctor’s office to disclose them. OSHA standards are established minimum health and safety standards for workers and provides for the inspection of places of employment and the penalizing of employers who do not provide conditions that meet the established standards, CLIA is legislation that promulgated quality assurance practices in clinical labs, and CDC Guidelines refers to all phases of control of communicable, vector-borne, and occupational diseases and with the prevention of disease, injury, and disability.

105
Q

Which of the following mandates how long patient records must be retained?

A. physician preference
B. federal and/or state laws
C. AMA guidelines
D. HIPAA regulations

A

B. federal and/or state laws

Rationale: Medicare and Medicaid, must be kept for 10 years. Federal and state regulations require a minimum storage time frame for medical records. State regulations may require a longer or shorter time frame, depending on the state. Every medical provider should routinely check state and federal requirements to remain compliant. As of the date this question was submitted (2015), a 7 to 10 year time frame would meet all federal and state medical record storage requirements. The AMA does not routinely mandate length of record storage. Physicians may choose to store records longer, but the actual mandates come from the state and federal levels. Patient privacy (HIPAA) will always need to be maintained, regardless of where or how long the records are stored (HIPAA does not issue mandates for length of storage).

106
Q

Which of the following describes a written request for reimbursement for eligible medical expenses?

A. superbill
B. CMS 1500
C. formulary
D. fee schedule

A

B. CMS 1500

Rationale: A CMS 1500 is initiated for insurance processing. A superbill is an itemized form utilized by healthcare providers for reflecting rendered services. A formulary is a list of prescription drugs covered by a health insurance plan. A fee schedule is a listing of the fees normally charged by a given health care provider for specific therapies and procedures provided.

107
Q

What does the abbreviation HIPAA stand for?

A. Health Information Privacy Adjustment Association
B. History Inquiry of Professional Assertive Assistance
C. Health Insurance Portability and Accountability Act
D. Health Inquiry for Permission of Accountability Association

A

C. Health Insurance Portability and Accountability Act

Rationale: HIPAA = Health Insurance Portability and Accountability Act. HIPAA (Health Insurance Portability and Accountability Act) requires medical professionals to protect the confidentiality of patients’ health information.

108
Q

Which of the following requires the patient’s signature just prior to being seen in the medical office?

A. consent for treatment
B. CMS-1500
C. EOB
D. DNR

A

A. consent for treatment

Rationale: Authorization for permission to treat is the legal right for patient care (this is a consent for treatment). A durable power of attorney for health care would be on file in the event the patient cannot made a medical decision if incapacitated in any way. DNR (do not resuscitate) is a legal order in which no CPR (cardiopulmonary resuscitation) or life support measures will be used on the patient. The CMS-1500 form is the official standard Medicare and Medicaid health insurance claim form and the CMS-1450 is also used for billing of institutional charges to most Medicaid State Agencies. EOB is an explanation of benefits that insurance carriers send the patient after treatment to explain what expenses were paid.

109
Q

Place the procedural steps below in the correct order for filing patients’ medical records. (Click and drag the options in the left column to their correct position in the right column).

A. sort
B. code
C. index
D. file

A

index, code, sort, file

Rationale: The medical assistant should first inspect the medical records, then determine the index and code. Next, sort the files for easier filing. The final step is placing the records in the proper files.

110
Q

Which of the following types of licensure must a physician have and maintain to legally dispense, prescribe, or administer controlled substances?

A. pharmaceutical
B. narcotics
C. business
D. occupational

A

B. narcotics

Rationale: Physicians must have a narcotics license in order to have the legal right to dispense controlled substances. The DEA maintains a database that is the primary source for DEA license verification. A pharmaceutical, business or occupational license does not grant a physician the legal right to dispense controlled substances.

111
Q

The medical assistant is completing front office opening tasks prior to the start of the day. Which of the following indicates the reception area is prepared to receive patients? (Select the three (3) correct answers.)

A. A throw rug is folded up at one corner.
B. Magazines are stacked on the side tables.
C. Sharps containers are placed by the entrance for biohazard pick-up.
D. Lights are turned on.
E. The path from the entrance to the front desk is direct and unobstructed.

A

B. Magazines are stacked on the side tables.
D. Lights are turned on.
E. The path from the entrance to the front desk is direct and unobstructed.

Rationale: Maintaining a clutter-free environment demonstrates professionalism and promotes safety in the medical office. Also making the environment attractive and “welcoming” is also part of concern for all patients. They perceive that the office is professional and inviting. Other activities are part of the rest of the daily work but not necessarily a first priority.

112
Q

Which of the following is a requirement for a healthcare facility to be in compliance with OSHA?

A. Payment of annual dues
B. Current certificate of compliance on file
C. Annual employee safety training
D. Annual inspections conducted by OSHA

A

C. Annual employee safety training

Rationale: MA’s in a Medical setting must be in compliance with OSHA guidelines, which include the following: a) A written exposure control plan, to be updated annually, Use of universal precautions, b) Consideration, implementation, and use of safer, engineered needles and sharps, c) Use of engineering and work practice controls and appropriate personal protective equipment (gloves face and eye protection, gowns), d) Hepatitis B vaccine provided to exposed employees at no cost, e) Medical follow-up in the event of an “exposure incident”, f) Use of labels or color-coding for items such as sharps disposal boxes and g) containers for regulated waste and contaminated laundry.

113
Q

The primary objective of which of the following agencies is to regulate and ensure quality of clinical laboratory testing?

A. OSHA
B. FDA
C. DEA
D. CLIA

A

D. CLIA

Rationale: CLIA – Clinical Laboratory Improvements Amendments of 1988 are United States federal regulatory standards that apply to all clinical laboratory testing performed on humans in the United States, except clinical trials and basic research. OSHA - Occupational Safety and Health Administration, a federal agency of the United States that regulates workplace safety and health. FDA - Food & Drug Administration is a federal agency of the United States Department of Health and Human Services. DEA - Drug Enforcement Administration is a United States federal law enforcement agency under the U.S. Department of Justice, tasked with combating drug smuggling and use within the United States.

114
Q

What is the best way to store paper charts?

A. On a book shelf in the office
B. In a file cabinet without a lock
C. In a file cabinet with a lock
D. In a desk drawer

A

C. In a file cabinet with a lock

Rationale: Paper medical records should be stored out of sight of unauthorized individuals. Paper medical records should be stored in a locked cabinet, room, office, closet or building when not supervised or in use.

115
Q

A medical assistant is hired at a Family Medical Practice. According to OSHA requirements the medical assistant must be offered a vaccine within 10 days of hire, free of charge. The medical assistant knows that this vaccination is to protect him from which of the following?

A. Tetanus
B. Varicella
C. Hepatitis B
D. Haemophilus influenza

A

C. Hepatitis B
Rationale: All health care employees are required to have access to the Hepatitis B injection. All employers must offer the series of injection with no cost to employees within 10 days of employment. Employees can decline their right to the vaccination, but must sign denial form.

116
Q

A large open container of isopropyl alcohol is spilled on the floor in the clinic waiting area. The medical assistant should use which of the following resources for information about the chemical and any necessary procedure?

A. Material Safety Data Sheet
B. Clinical Laboratory Improvement Amendments
C. Poison Control Center
D. Physician’s Desk Reference

A

A. Material Safety Data Sheet

Rationale: Access to Material Safety Data Sheets is an OSHA compliance requirement. Each MSDS contains information on the potential hazards (health, fire, reactivity and environmental) and how to work safely with chemical products. It is an essential starting point for the development of a complete health and safety program. Clinical Laboratory Improvement Amendments (CLIA) of 1988 are United States federal regulatory standards that apply to all clinical laboratory testing performed on humans in the United States, except clinical trials and basic research. A poison control center is a medical facility that is able to provide immediate, free, and expert treatment advice and assistance over the telephone in case of exposure to poisonous or hazardous substances. The purpose of the Physician’s Desk Reference is to provide a compendium compiled annually, containing information supplied by drug manufacturers about drugs, primarily prescription drugs and products used in diagnostic procedures in the United States. A Material Safety Data Sheet includes information such as physical, health, and environmental health hazards; protective measures; and safety precautions for handling, storing, and transporting chemicals including all safe handling practices, and emergency control measures in the medical office.

117
Q

Which of the following is necessary before ordering or paying for medical office supplies?

A. Purchase order
B. Inventory list
C. Packing slip
D. Order number

A

A. Purchase order

Rationale: A purchase order is required for authorization and expense documentation of transaction. An inventory list may have been used to determine what supplies needed to be ordered, and would be updated with the supplies received. A packing slip would be included within the order shipment to show what items were “packed” in the box or envelope received. Companies generally issue an order number to help track which order is associated with which items, which client, etc.

118
Q

Which of the following requires the medical office to develop an exposure control plan?

A. FDA
B. CLIA
C. CDC
D. OSHA

A

D. OSHA

Rationale: An OSHA (Bloodborne Pathogens Standard) Exposure Control Plan, must meet certain criteria – It must be reviewed and updated at least yearly and be readily available to all medical staff. Clinical Laboratory Improvement Amendments (CLIA) are federal regulatory standards that pertain to laboratory testing on human specimens.The Food and Drug Administration (FDA) and the Centers for Disease Control and Prevention (CDC) do not mandate an exposure control plan.

119
Q

Which of the following statements is true about the SDS?

A. The SDS contains basic information about specific chemicals.
B. SDS must be kept on file for 30 days.
C. OSHA is responsible for developing the hazard identification system.
D. The SDS is required only in offices with 10 or more employees.

A

A. The SDS contains basic information about specific chemicals.

Rationale: A safety data sheet (SDS) must be on file for all chemicals used in the laboratory. OSHA requires the manufacturer of the chemical to make these sheets available, usually as a package insert or online. Employers must ensure that SDSs are readily accessible to employees. SDSs supply information about chemical safety and hazard information in case of an accident, spill, or fire to the user personnel.

120
Q

A medical assistant is reviewing a patient’s encounter that is documented in the medical record prior to completing a CMS-1500 form. She notices that the physician upcoded the encounter. The medical office assistant has the ethical obligation to

A. change the code.
B. debate the code.
C. alter the code.
D. down code the code.

A

B. debate the code.

Rationale: It is within the medical assistant’s ethical responsibility to debate the code. It is illegal to purposely “upcode” an encounter for any reason. The coding system is specific and should be followed. The office assistant does not have the authority to change, alter, or down code the code without consulting the physician.

121
Q

Which of the following organizations makes on-site visits to inspect medical office laboratories?

A. WHO
B. DEA
C. CLIA
D. FDA

A

C. CLIA

Rationale: All laboratory testing in the United States that is performed on humans is highly regulated. The Centers for Medicare & Medicaid Services (CMS) governs laboratories via the Clinical Laboratory Improvement Amendments (CLIA).

122
Q

Which of the following is the most likely cause of the deposits not agreeing with the credits on the day sheet or the patient ledgers?

A. There are duplicate cards.
B. The bank made an error.
C. Cash is missing.
D. Payment is misplaced.

A

D. Payment is misplaced.

Rationale: The first step of reconciliation is to first determine if a payment is misplaced. Then issues of duplication or misplaced monies can be addressed. Transactions involving a possible bank error would be the last thing to check before checking the medical office records.

123
Q

A patient presents to the provider’s office with a complaint of persistent migraines three days after acquiring a head injury on the job. After the provider assesses the patient, the final diagnosis is a concussion. Which of the following actions should the medical assistant take next?

A. File a claim with the patient’s primary insurance carrier.
B. Bill the patient’s employer directly.
C. Determine if a workman’s compensation claim has been filed.
D. Obtain payment directly from the patient at the time of service.

A

C. Determine if a workman’s compensation claim has been filed.

Rationale: Whenever a patient claims a workplace injury, the health care provider should call the employer to see if a workman’s compensation claim has been filed. If so, billing will first go through that claim. The other options listed in this scenario would be follow-up based on whether or not a workman’s comp claim has been filed.

124
Q

A patient who is insured by an HMO calls the office stating he received a bill of $185. His deductible has already been met and he made the copayment of $10 at the time of service. The HMO paid 80% of the charges. Which of the following parties is responsible for the remaining balance?

A. the physician
B. the patient
C. a third party payer
D. a PPO

A

B. the patient

Rationale: If the patient has an 80/20 policy, then the insurance (third party payer) is responsible for 80% and the patient is responsible for the remaining 20%, this is known as coinsurance. If the patient carries a secondary insurance, that secondary insurance will be billed after the primary (first) insurance has determined the amount that will pay out. This is known as coordination of benefits. Premiums, deductibles, copayments, and coinsurance balances are the responsibility of the patient.

125
Q

The medical assistant adheres to which of the following provisions to ensure the patient with multiple insurance plans is not reimbursed more than once for care received?

A. coordination of benefits
B. coordination of care
C. preauthorization
D. explanation of benefits

A

A. coordination of benefits

Rationale: There are established rules to determine the primary carrier (billed first), secondary carrier, etc., when a patient is covered by more than one insurance company or when dependents are involved. The purpose of the provision is to coordinate benefits, so the total amount of benefits paid does not exceed the total amount of expenses incurred. The birthday rule is a good example. When children are dependents on each parent’s group health care plan, the primary insurance will be the parent with a birthday earliest in the calendar year.

126
Q

Which of the following prevents duplication of payment by more than one insurance carrier?

A. Fee for service
B. Precertification
C. Coordination of benefits
D. Preauthorization

A

C. Coordination of benefits

Rationale: Coordination of benefits is needed when a patient is covered by one or more plan (i.e. insurance and Medicare) to determine how much and for which services each plan is responsible for paying. The primary insurance carrier must be determined (and billed first) to ensure record accuracy. Precertification is the process of finding out if a service or procedure is covered under a patient’s insurance policy. Once it’s determined that a procedure/service is covered, permission (preauthorization) must be obtained from the insurance provider. Fee-for-service (FFS) delivery systems pay providers for each service rendered. For instance, a separate fee will be paid for the office visit, lab tests, x-ray, etc.

127
Q

Which of the following resources should the medical assistant use to look up the code for a laceration repair?

A. International Classification of Disease
B. Physician’s Desk Reference
C. Healthcare Common Procedural Coding System
D. Current Procedural Terminology

A

D. Current Procedural Terminology

Rationale: A laceration repair would be found in the CPT code book. Current Procedural Terminology identifies services and procedures. International Classification of Diseases (ICD) identifies diagnoses. Healthcare Common Procedure Coding System (HCPCS) – identifies additional services and procedures not listed in CPT; drugs dispensed in the physician’s office or other facility (including injectables, topicals, and orally administered); supplies and equipment dispensed to the patient. The Physician’s Desk Reference is a publication that compiles drug information for medical professionals and consumers alike.

128
Q

Which part of Medicare covers office visits with a primary care provider?

A. Part A
B. Part B
C. Part C
D. Part D

A

B. Part B

Rationale: A monthly premium and meeting an annual deductible are standard to qualify, after which an 80% reimbursement rate will take effect - the insured is responsible for the 20% coinsurance. [Part A - hospital insurance, Part B - helps to offset costs not covered by Part A, Part C – Medicare Advantage, allows beneficiaries to select a managed care plan as their primary coverage Part D – prescription drug coverage, helps with coverage of some medications.]

129
Q

The provider prescribed and ordered a wheel chair for a patient with a below the knee amputation. Which of the following manuals should the medical assistant use to code these services?

A. ICD-10-CM
B. CPT
C. HCPCS
D. CPT-assistant

A

C. HCPCS

Rationale: A wheel chair is classified as durable medical equipment. A prescription for a wheel chair would be coded for in the HCPCS. Healthcare Common Procedure Coding System (HCPCS) level II codes address durable medical equipment and other services not in the level I Current Procedural Terminology (CPT) codes. ICD (International Classification of Disease) codes are associated with the diagnosis/disease instead of procedures.

130
Q

The patient is referred to a neurosurgeon. Entering the specialist’s NPI number and number of visits requested into the patient’s insurance carrier’s website is part of which of the following processes?

A. Precertification
B. Compensation
C. Electronic claims submission
D. Assignment of benefits

A

A. Precertification

Rationale: This is an example of the precertification process, which facilitates prior authorization to confirm eligibility for treatment. Precertification is a process to establish medical necessity before a treatment is approved. Once precertified, the patient can then see the neurosurgeon. The patient will sign an assignment of benefits form to allow insurance to directly pay the medical provider. Then a claim will be submitted (most likely electronically) and insurance will pay the claim. Some companies offer worker compensation benefits for employees who are injured on the job.

131
Q

The patient is referred to a neurosurgeon. Entering the specialist’s NPI number and number of visits requested into the patient’s insurance carrier’s website is part of which of the following processes?

A. Precertification
B. Compensation
C. Electronic claims submission
D. Assignment of benefits

A

A. Precertification

Rationale: This is an example of the precertification process, which facilitates prior authorization to confirm eligibility for treatment. Precertification is a process to establish medical necessity before a treatment is approved. Once precertified, the patient can then see the neurosurgeon. The patient will sign an assignment of benefits form to allow insurance to directly pay the medical provider. Then a claim will be submitted (most likely electronically) and insurance will pay the claim. Some companies offer worker compensation benefits for employees who are injured on the job.

132
Q

While a new patient is in the examination room with a physician who is explaining treatment options to the patient, the medical assistant is contacting the insurance carrier to discuss the patient’s insurance coverage. This scenario is an example of obtaining

A. a preauthorization for the patient.
B. a statement of probable cause.
C. a disclosure of benefits and eligibility for the patient.
D. verification of benefits and eligibility for the patient.

A

D. verification of benefits and eligibility for the patient.

Rationale: It is important to verify what benefits and eligibility the patient has under the insurance policy. This will assist all involved in determining the next course of action. If there are options for treatment, the patient and provider need to know what is and what is not covered before choosing a treatment plan. A preauthorization, probable cause, or referral may be indicated, but the first thing to determine is the patients benefits and eligibility, then follow up accordingly.

133
Q

A parent brings their four-year old in for a well-child exam. The medical assistant should assign an ICD-CM code beginning with which of the following?

A. E
B. M
C. V
D. +

A

C. V

Rationale: V codes are used to indicate an encounter with no current illness or injury. Depending on the medical situation, V codes can be the primary (listed first) or secondary (contributing) code. E/M coding deals with evaluation and management. E codes are considered supplemental. They are used to list an external cause (i.e. what caused the injury). “+” is not a commonly used symbol in the medical coding system.

134
Q

The primary care provider hands the medical assistant a referral for a patient to get an MRI. Which of the following should the medical assistant do first?

A. Call the facility and schedule the MRI.
B. Call the insurance company and process an authorization.
C. Fax pertinent medical records to the facility.
D. Document the precertification number in the patient’s chart.

A

B. Call the insurance company and process an authorization.

Rationale: An MRI can be a pricey procedure. Patients definitely don’t want to pay any more than they should. A medical assistant should call the insurance company and get an authorization before sending the patient for the procedure. Once authorization is obtained, then the precertification number should be documented on the patient’s chart. The MRI facility can then be called to schedule the procedure; patient chart information can be faxed, if necessary, at that time.

135
Q

A patient has refused to pay for a medical procedure that was performed six months ago. The medical procedure was not listed under the patient’s schedule of benefits, and she is now fully responsible for all costs. Her account has now been turned over to a collection agency. This scenario is most likely an example of failure of the medical office assistant to properly

A. code the procedure.
B. bill the procedure.
C. explain the non-coverage billing policies.
D. explain the statute of limitations.

A

C. explain the non-coverage billing policies.

Rationale: Making sure that the office has a policy in place to let your patients know what you expect of them and what they can expect of you. A well-crafted policy will prevent patients from being surprised about their financial obligation when they receive billing for your services. It will also give your practice some legal protection should a patient fail to pay what you are entitled to collect. The policy should be tailored to the medical office policy and practice. Being up front with the patients prevents future error or misunderstandings.

136
Q

A Medicare patient presents to an outpatient hospital facility for a scheduled hysterectomy. To which Medicare plan should the facility submit the claim?

A. Medicare Part A
B. Medicare Part B
C. Medicare Part C
D. Medicare Part D

A

B. Medicare Part B

Rationale: This claim should be submitted to Medicare Part B since it is an outpatient medical procedure. Medicare Part A basically covers inpatient care, but can expand to cover hospice and home health in a limited capacity. Medicare Part B is generally referred to as medical insurance that covers outpatient visits, services and supplies. Medicare Part C (a.k.a. Medicare Advantage) is an additional option for private insurance on top of A and B. Medicare Part D is associated with prescription drugs.

137
Q

An established patient who has Chronic Obstructive Pulmonary Disease is evaluated in the medical office for hypertension. What is the importance of identifying the reason for today’s visit related to billing?

A. To determine the primary diagnosis
B. To ensure a secondary diagnosis is not listed on the claim form
C. To assign a two-digit modifier
D. To assign a V code

A

A. To determine the primary diagnosis

Rationale: In this example, hypertension should be listed as the primary diagnosis (main reason the patient sought care for visit) and COPD as secondary (this will disease will be considered when medications are prescribed for HTN.)

138
Q

A medical assistant is reviewing a chart with the following documentation: patient presented with a complaint of itchy, red bumps on her chest and neck. Diagnosis: Urticaria, Procedure: Expanded Office Visit. The coding reference manual that would contain the term Urticaria and the associated code is the

A. Current Procedural Terminology (CPT) code book.
B. Health Care Financing Administration Common Procedure Coding System (HCPCS) code book.
C. Centers for Medicare and Medicaid Services (CMS) code book.
D. International Classification of Diseases (ICD) code book.

A

D. International Classification of Diseases (ICD) code book.

Rationale: Urticaria is a type of skin rash commonly known as hives. In the scope of medicine, this diagnosis is considered the “disease” and would be found in the International Classification of Disease (ICD) reference manual. CPT and HCPCS are both associated with procedures. CMS is the Centers for Medicare & Medicaid Services and requires forms to be filled out (to include applicable ICD, CPS and HCPCS codes).

139
Q

The patient presents to the pre-op clinic for a lithotripsy (ESWL) procedure. Which of the following actions should the medical assistant take prior to witnessing the consent form to ensure it is legally binding?

A. Verify the patient has not had anything to eat or drink for eight hours prior to the procedure.
B. Obtain a copy of the patient’s insurance card.
C. Confirm there is someone in the waiting room available to drive the patient home.
D. Confirm the surgeon has explained all risks and benefits of the procedure.

A

D. Confirm the surgeon has explained all risks and benefits of the procedure.

Rationale: The medical assistant has an ethical obligation to make sure the surgeon has done what is legally required of him/her and has explained all of the risks and benefits of the procedure to the patient. The other options are all important to do as well, but the priority action to ensure the consent form is legally binding is to confirm that the surgeon explained the risks and benefits.

140
Q

A husband and wife enter the medical office for a visit. The wife is called back and the husband makes the decision to stay in the waiting room. The husband proceeds to ask the medical assistant when his wife’s next appointment is. Which of the following is the correct response from the medical assistant?

A. “Your wife’s next appointment is in 3 months.”
B. “I’m sorry I cannot release that information.”
C. “Let me check with the doctor.”
D. “She doesn’t have a follow-up would you like to schedule?”

A

B. “I’m sorry I cannot release that information.”

Rationale: If the medical assistant does not have the chart in front of them, they cannot release that information without knowing if there is written permission to discuss patient information with the husband. A patent must give permission in writing or verbal permission if the spouse is in office, to have any health information released to someone other than the patient. This must be kept in the patient’s chart.

141
Q

A patient whose treatment plan includes chemotherapy and radiation asked the medical assistant to provide information on alternative treatments. Which of the following actions should the medical assistant take?

A. Research alternative treatment methods and provide the results to the patient.
B. Recommend alternative treatment literature to the patient.
C. Direct the patient to websites and other alternative treatment resources.
D. Tell the patient to discuss alternative treatment methods with the physician.

A

D. Tell the patient to discuss alternative treatment methods with the physician.

Rationale: This is out of the scope of practice of a medical assistant. Under the doctrine of Informed Consent, the provider, NOT the medical assistant has a duty to provide information to patient or guardian. The provider must educate the patient on treatment options.

142
Q

An incorrect dose of medication is delivered to a patient in the medical clinic. As a result, the patient experienced mild kidney damage. The responsible health care providers and/or clinic is at risk for which of the following?

A. Slander
B. Libel
C. Defamation
D. Negligence

A

D. Negligence

Rationale: This is a case of negligence, a failure to practice within the standard of care. Defamation occurs when someone intentionally says or writes something false that causes harm to the person’s reputation. Slander is a form of defamation that is heard/spoken. Libel is a form of defamation that is visible (written, produced, etc.).

143
Q

Which of the following components are included in the “4 D’s” of negligence? (Select the four (4) correct answers.)

A. Denial
B. Damages
C. Duty
D. Direct cause
E. Dereliction

A

B. Damages
C. Duty
D. Direct cause
E. Dereliction

Rationale: Damages – award or compensation to be paid. Duty – obligation to provide care in established physician-patient relationship. Direct cause – proof of direct or indirect harm. Dereliction - failure to perform duty.

144
Q

Failure to act as the law obligates you to act, within the expected standard of care, is known as

A. misfeasance.
B. breach of duty.
C. malfeasance.
D. abuse.

A

B. breach of duty.

Rationale: Breach of duty is the failure to act as the law requires within the expected standard of care. Abuse, on the other hand, would be indicated if the medical personnel acted in a manner to intentionally cause harm or injury to a patient. Misfeasance is willful, inappropriate action or intentional incorrect action or advice, and malfeasance is willful and intentional action that injures someone.

145
Q

Which of the following is likely considered outside the scope of practice for a medical assistant?

A. Verify equipment functionality using a control material
B. Perform additional tests on different instruments for validation
C. Record quality control results on a log sheet
D. Perform CLIA high complexity tests

A

D. Perform CLIA high complexity tests

Rationale: CLIA refers to the Clinical Laboratory Improvement Amendment, which specifies personnel requirements for performing laboratory testing based upon their complexity. Moderate and high complexity testing are outside the scope of practice for a phlebotomist or MA. Medical assistants may perform CLIA-waived testing, such as Point of Care Testing, with proper training.

146
Q

Which of the following tasks is outside the scope of practice of a medical assistant?

A. Using an ECG to record the electrical rhythm of the heart
B. Performing patient education related to diabetes
C. Dispensing medication, such as Glucola, in collaboration with the physician
D. Performing quality control functions on all laboratory equipment

A

D. Performing quality control functions on all laboratory equipment

Rationale: The quality control on equipment and supplies commonly used in phlebotomy is typically performed by a medical assistant, but major equipment is quality controlled by credentialed laboratory professionals. Other healthcare professionals would perform tasks within their own scope of practice. ECG technicians and MAs would perform ECG recordings. Dietitians would perform nutritional assessments, though MAs might assist with patient education related to diabetes (under the direction of the dietitian or physician). MAs could dispense medication upon physician orders, always following proper administration protocol.

147
Q

Which of the following tasks is outside the scope of practice of a medical assistant?

A. Using an ECG to record the electrical rhythm of the heart
B. Performing patient education related to diabetes
C. Dispensing medication, such as Glucola, in collaboration with the physician
D. Performing quality control functions on all laboratory equipment

A

D. Performing quality control functions on all laboratory equipment

Rationale: The quality control on equipment and supplies commonly used in phlebotomy is typically performed by a medical assistant, but major equipment is quality controlled by credentialed laboratory professionals. Other healthcare professionals would perform tasks within their own scope of practice. ECG technicians and MAs would perform ECG recordings. Dietitians would perform nutritional assessments, though MAs might assist with patient education related to diabetes (under the direction of the dietitian or physician). MAs could dispense medication upon physician orders, always following proper administration protocol.

148
Q

Commitment to privacy, continuity of care, advance directives, and the authority to refuse treatment are granted according to which of the following?

A. Americans with Disabilities Act
B. The Patient’s Bill of Rights
C. Informed Consent
D. Good Samaritan Law

A

B. The Patient’s Bill of Rights

Rationale: Patients are guaranteed certain standards of care listed in this question by The Patient’s Bill of Rights. In 2010, in conjunction with the Affordable Care Act, a new Patient’s Bill of Rights was set forth to include protections associated with insurance companies. Patients do have a right to be fully informed of a procedure or treatment option and have the right to consent to it or refuse. The Americans with Disabilities Act (ADA) addresses discrimination of disabled individuals. The Good Samaritan Law protects individuals who are willing to help someone in an emergency situation.

149
Q

A patient is unhappy because the physician refused to prescribe more Percocet. The patient tells other patients the doctor is incompetent and will not provide proper medical care. Which of the following could the patient be charged with if the physician proves the statement is untrue or the patient damaged his reputation?

A. Libel
B. Malpractice
C. Slander
D. Negligence

A

C. Slander

Rationale: The patient could be charged with slander: making an untrue statement that can hurt someone’s reputation. Libel is very similar to slander, however, the false statement is printed and not just stated. Malpractice is an illegal or negligent act done typically by a physician, public official, or lawyer. Negligence occurs when someone is careless when caring for another person and fails to perform an action that is expected of them, therefore resulting in injury or damage.

150
Q

Which of the following acts involves the termination of a patient from the care of a physician without reasonable, written notice of withdrawal from the case?

A. Slander
B. Discrimination
C. Battery
D. Abandonment

A

D. Abandonment

Rationale: A physician or primary care provider who wishes to end a patient-physician relationship must follow a formal notification process to avoid a claim of patient abandonment. The medical assistant may help with this process to ensure proper completion. Steps include writing a formal notification letter informing the patient, including reason(s) for withdrawal and effective time frame, mailing the letter certified with return receipt, ensuring a copy of the letter and all documentation is placed in the patient’s medical record, documenting the termination in the patient’s record and actions taken.

151
Q

During a routine exam, the physician determines that the patient needs to have an immediate I & D, to be performed in the office. The physician explains the procedure to the patient and asks the medical office assistant to obtain consent and record the proper documentation. Upon completion of this task, the medical office assistant alerts the doctor that the patient is ready for the procedure. This scenario is an example of

A. implied consent.
B. informed consent.
C. verbal consent.
D. expressed consent.

A

**B. informed consent.
**

Rationale: This is a scenario of informed consent. The patient agrees to and signs a document in regards to a procedure after the provider explains the risks and consequences. Implied consent means consent is understood by the patient’s actions even though the patient did not directly express consent. A patient agreeing to a procedure even though it is not written down in contract is considered verbal consent and expressed consent comes when the patient gives permission to the procedure either verbally or non-verbally.

152
Q

Which of the following refers to a physician’s written order specifying a patient has indicated that he or she does not want CPR?

A. living will
B. durable power of attorney for health care
C. Advanced Beneficiary Notice
D. DNR

A

D. DNR

Rationale: A DNR (do not resuscitate) order is generated at the request of a patient, or by the patient’s durable power of attorney for health care if the patient is incapable of making medical decisions. A DNR indicates that cardiopulmonary resuscitation (CPR) should not be performed to sustain life in the event that the patient is in a medical crisis. A patient’s advanced directive (a.k.a. living will) is also kept on file with the electronic health record or chart and includes information regarding the patient’s values, goals for treatment, and wishes that should be considered if the patient is incapable of making those medical decisions. An advanced beneficiary notice of noncoverage (ABN) is a form that patients sign to accept payment responsibility for tests/procedures that may not be covered by Medicare.

153
Q

Which of the following indicates a medical office is compliant with the Americans with Disabilities Act?

A. Providing interpreting services for a non-English speaking patient.
B. Providing employees the opportunity to receive the Hepatitis B vaccine.
C. Providing hand rails in at least one restroom stall.
D. Providing up to 12 weeks of unpaid leave to an employee that has a critically ill child.

A

C. Providing hand rails in at least one restroom stall.

Rationale: By providing access and accessibility for individuals with disabilities, the Americans with Disabilities Act secures each person’s right to an independent life. Providing a bi-lingual service would be only a courtesy by the provider for non-English speaking patients. Providing services for employees is not covered under the ADA.

154
Q

What is the term that refers to the requirements and limits that a healthcare professional must work within?

A. Duty to act
B. Limited contract
C. Good Samaritan Act
D. Scope of practice

A

D. Scope of practice

Rationale: Duty to act refers to duty of a party to take action to prevent harm to a party or the general public. The Good Samaritan Act refers to a person that renders aid in an emergency on a voluntary basis. A limited contract is an employment agreement measured by time and renews when it expires. Regulatory agencies set forth educational requirements and limits that a healthcare professional must work within, called the scope of practice.

155
Q

A medical assistant posts damaging remarks on a social media site about a provider at the clinic where she is employed. She may be legally found guilty of which of the following?

A. Assault
B. Libel
C. Slander
D. Battery

A

B. Libel

Rationale: This would be an example of libel, which is the publishing of an accusation or false statement which can cause harm to a person or ruin that person’s reputation. It is an example of an intentional tort. Slander is when a false accusation or statement is spoken about someone else with effects similar to libel. Assault is a threat to cause harm to another person, whereas battery is the physical harm one person causes another.